Re: The Nature of Contingency: Quantum Physics as Modal Realism

78 views
Skip to first unread message

Brent Meeker

unread,
Apr 28, 2022, 1:24:13 AM4/28/22
to Everything List


On 4/26/2022 5:32 PM, smitra wrote:
On 27-04-2022 01:37, Bruce Kellett wrote:
On Tue, Apr 26, 2022 at 10:03 AM smitra <smi...@zonnet.nl> wrote:

On 24-04-2022 03:16, Bruce Kellett wrote:

A moment's thought should make it clear to you that this is not
possible. If both possibilities are realized, it cannot be the
case
that one has twice the probability of the other. In the long run,
if
both are realized they have equal probabilities of 1/2.

The probabilities do not have to be 1/2.  Suppose one million people

participate in a lottery such that there will be exactly one winner.
The
probability that one given person will win, is then one in a
million.
Suppose now that we create one million people using a machine and
then
organize such a lottery. The probability that one given newly
created
person will win is then also one in a million. The machine can be
adjusted to create any set of persons we like, it can create one
million
identical persons, or almost identical persons, or totally different

persons. If we then create one million almost identical persons, the

probability is still one one in a million. This means that the limit
of
identical persons, the probability will be one in a million.

Why would the probability suddenly become 1/2 if the machine is set
to
create exactly identical persons while the probability would be one
in a
million if we create persons that are almost, but not quite
identical?

Your lottery example is completely beside the point.

It provides for an example of a case where your logic does not apply.

I think you
should pay more attention to the mathematics of the binomial
distribution. Let me explain it once more: If every outcome is
realized on every trial of a binary process, then after the first
trial, we have a branch with result 0 and a branch with result 1.
After two trials we have four branches, with results 00, 01, 10,and
11; after 3 trials, we have branches registering 000, 001, 011, 010,
100, 101, 110, and 111. Notice that these branches represent all
possible binary strings of length 3.

After N trials, there are 2^N distinct branches, representing all
possible binary sequences of length N. (This is just like Pascal's
triangle) As N becomes very large, we can approximate the binomial
distribution with the normal distribution, with mean 0.5 and standard
deviation that decreases as 1/sqrt(N). In other words, the majority of
trials will have equal, or approximately equal, numbers of 0s and 1s.
Observers in these branches will naturally take the probability to be
approximated by the relative frequencies of 0s and 1s. In other words,
they will take the probability of each outcome to be 0.5.


The problem with this is that you just assume that all branches are equally probable. You don't make that explicit, it's implicitly assumed, but it's just an assumption. You are simply doing branch counting.

But it shows why you can't use branch counting.  There's no physical mechanism for translating the a and b of  |psi> = a|0> + b|1> into numbers of branches.  To implement that you have put it in "by hand" that the branches have weights or numerousity of a and b.  This is possible, but it gives the lie to the MWI mantra of "It's just the Schroedinger equation."

Brent



The important point to notice is that this result of all possible
binary sequences for N trials is independent of the coefficients in
the binary expansion of the state:

     .

Changing the weights of the components in the superposition does not
change the conclusion of most observers that the actual probabilities
are 0.5 for each result. This is simple mathematics, and I am amazed
that even after all these years, and all the times I have spelled this
out, you still seek to deny the obvious result. Your logical and
mathematical skill are on a par with those of John Clark.


It's indeed simple mathematics. You apply that to branch counting to arrive at the result of equal probabilities. So, the conclusion has to be that one should not do branch counting. The question is then if this disproves the MWI. If by MWI we mean QM minus collapse then clearly not. Because in that case we use the Born rule to compute the probability of outcomes and assume that after a measurement we have different sectors for observers who have observed the different outcomes with the probabilities as given by the Born rule.

You then want to argue against that by claiming that your argument applies generally and would not allow one to give different sectors unequal probabilities. But that's nonsense, because you make the hidden assumption of equal probabilities right from the start. There is nothing in QM that says that branches must count equally, and the lottery example I gave makes it clear that you can have branching with unequal probabilities in classical physics.

Yes, there's nothing in QM that says the branches must count equally.  But there's also nothing in the evolution of Schroedingers equation that they must count as a^2 and b^2.  Of course IF they are probabilities then it follows from Gleason's theorem that they follow the Born rule.  But in that case you have reintroduced almost all the philosophical problems of the Copenhagen interpretation.  When exactly does this splitting occur?  Can the split be into irrational numbers of branches?  A splitting is in some particular basis and not in other bases.  What determines the pointer basis?

Brent


Saibal



Bruce

 --
You received this message because you are subscribed to the Google
Groups "Everything List" group.
To unsubscribe from this group and stop receiving emails from it, send
an email to everything-li...@googlegroups.com.
To view this discussion on the web visit
https://groups.google.com/d/msgid/everything-list/CAFxXSLT22raNMhxUsrUqHni2P-T4Ww%3DXQh_HKUO7CBpTZv8q_Q%40mail.gmail.com
[1].


Links:
------
[1]
https://groups.google.com/d/msgid/everything-list/CAFxXSLT22raNMhxUsrUqHni2P-T4Ww%3DXQh_HKUO7CBpTZv8q_Q%40mail.gmail.com?utm_medium=email&utm_source=footer


Bruce Kellett

unread,
Apr 28, 2022, 1:51:55 AM4/28/22
to Everything List
On Thu, Apr 28, 2022 at 3:24 PM Brent Meeker <meeke...@gmail.com> wrote:
On 4/26/2022 5:32 PM, smitra wrote:
On 27-04-2022 01:37, Bruce Kellett wrote: 
Changing the weights of the components in the superposition does not
change the conclusion of most observers that the actual probabilities
are 0.5 for each result. This is simple mathematics, and I am amazed
that even after all these years, and all the times I have spelled this
out, you still seek to deny the obvious result. Your logical and
mathematical skill are on a par with those of John Clark.

It's indeed simple mathematics. You apply that to branch counting to arrive at the result of equal probabilities.
 
I have not used branch counting. Please stop accusing me of that.
So, the conclusion has to be that one should not do branch counting. The question is then if this disproves the MWI. If by MWI we mean QM minus collapse then clearly not. Because in that case we use the Born rule to compute the probability of outcomes and assume that after a measurement we have different sectors for observers who have observed the different outcomes with the probabilities as given by the Born rule.

In which case the Born rule is just an additional arbitrary assumption: it is not part of the Schrodinger equation. Your theory of QM minus collapse is not well-defined. You simply take whatever you want from text-book quantum mechanics, with no regard to the consistency of your model.
You then want to argue against that by claiming that your argument applies generally and would not allow one to give different sectors unequal probabilities. But that's nonsense, because you make the hidden assumption of equal probabilities right from the start.

I simply assume the Schrodinger equation. Then, following Everett, we take it to be deterministic, so that all branches occur on every trial. Since it is deterministic, there is no concept of probability inherent in the Schrodinger equation, and I do not assume any definition of probability. So the branches occur as they occur, there is no assumption of equal probability. It is just that the construction means that  all 2^N branches occur on the same basis and necessarily count equally in the overall branching picture.

There is nothing in QM that says that branches must count equally, and the lottery example I gave makes it clear that you can have branching with unequal probabilities in classical physics.

As I have said, there is no classical analogue of an interaction in which all outcomes necessarily occur. So your lottery example is useless. There is no concept of probability involved in any of this.

Bruce

smitra

unread,
May 3, 2022, 8:01:32 AM5/3/22
to everyth...@googlegroups.com
> mechanism for translating the _a_ and _b_ of _|psi> = a|0> + b|1>_
> into numbers of branches. To implement that you have put it in "by
> hand" that the branches have weights or numerousity of _a _and _b_.
> This is possible, but it gives the lie to the MWI mantra of "It's just
> the Schroedinger equation."
>

Yes, one has to interpret the wavefunction as giving probabilities.
That's still better than assuming that the physical state evolves
sometimes according to the Schrödinger equations and sometimes by
undergoing a nondeterministic collapse without there being any evidence
for such collapses, without even credible theoretical models for it.
> that they must count as _a^2_ and _b^2_. Of course IF they are
> probabilities then it follows from Gleason's theorem that they follow
> the Born rule. But in that case you have reintroduced almost all the
> philosophical problems of the Copenhagen interpretation. When exactly
> does this splitting occur? Can the split be into irrational numbers
> of branches? A splitting is in some particular basis and not in other
> bases. What determines the pointer basis?
>

Pointer basis should be defined by defining observers as algorithms. If
we take Mr. Data from Star Trek as an example., All the states where he
has definite experiences be described as bistrings where the registers
are in well defined states. These are then the pointer states, they
define the set of all possible observations Mr. Data can make.

Saibal

> Brent
>

smitra

unread,
May 3, 2022, 8:11:57 AM5/3/22
to everyth...@googlegroups.com
On 28-04-2022 07:51, Bruce Kellett wrote:
> On Thu, Apr 28, 2022 at 3:24 PM Brent Meeker <meeke...@gmail.com>
> wrote:
>
>> On 4/26/2022 5:32 PM, smitra wrote:
>>
>>> On 27-04-2022 01:37, Bruce Kellett wrote:
>> Changing the weights of the components in the superposition does
>> not
>> change the conclusion of most observers that the actual
>> probabilities
>> are 0.5 for each result. This is simple mathematics, and I am amazed
>>
>> that even after all these years, and all the times I have spelled
>> this
>> out, you still seek to deny the obvious result. Your logical and
>> mathematical skill are on a par with those of John Clark.
>>
>> It's indeed simple mathematics. You apply that to branch counting to
>> arrive at the result of equal probabilities.
>
> I have not used branch counting. Please stop accusing me of that.
>

You are considering each branch to have an equal probability when there
is no logical reason to do so, and when that's also being contradicted
by QM.
'
>>> So, the conclusion has to be that one should not do branch
>>> counting. The question is then if this disproves the MWI. If by
>>> MWI we mean QM minus collapse then clearly not. Because in that
>>> case we use the Born rule to compute the probability of outcomes
>>> and assume that after a measurement we have different sectors for
>>> observers who have observed the different outcomes with the
>>> probabilities as given by the Born rule.
>
> In which case the Born rule is just an additional arbitrary
> assumption: it is not part of the Schrodinger equation. Your theory of
> QM minus collapse is not well-defined. You simply take whatever you
> want from text-book quantum mechanics, with no regard to the
> consistency of your model.
>

QM includes the Born rule. QM minus collapse is just that: QM minus
collapse. It's not QM minus collapse minus the Born rule.


>>> You then want to argue against that by claiming that your argument
>>> applies generally and would not allow one to give different
>>> sectors unequal probabilities. But that's nonsense, because you
>>> make the hidden assumption of equal probabilities right from the
>>> start.
>
> I simply assume the Schrodinger equation. Then, following Everett, we
> take it to be deterministic, so that all branches occur on every
> trial. Since it is deterministic, there is no concept of probability
> inherent in the Schrodinger equation, and I do not assume any
> definition of probability. So the branches occur as they occur, there
> is no assumption of equal probability. It is just that the
> construction means that all 2^N branches occur on the same basis and
> necessarily count equally in the overall branching picture.
>

Why do they necessarily count equally? What is the meaning of the
wavefunction? Why don't the amplitudes matter?


>>> There is nothing in QM that says that branches must count equally,
>>> and the lottery example I gave makes it clear that you can have
>>> branching with unequal probabilities in classical physics.
>
> As I have said, there is no classical analogue of an interaction in
> which all outcomes necessarily occur. So your lottery example is
> useless. There is no concept of probability involved in any of this.
>

The lottery example I gave clearly is a classical example in which all
outcomes necessarily occur. Your reasoning does not involve any QM at
all, you just apply it to the MWI. Your argument goes through also in
case of the lottery example, in which case it leads to an obviopusly
wrong conclusion. So, it's your reasoning that's at fault not the MWI
taken to be QM minus collapse.

Saibal

> Bruce
>
>>>
>>
>> Yes, there's nothing in QM that says the branches must count
>> equally. But there's also nothing in the evolution of Schroedingers
>> equation that they must count as _a^2_ and _b^2_. Of course IF they
>> are probabilities then it follows from Gleason's theorem that they
>> follow the Born rule. But in that case you have reintroduced almost
>> all the philosophical problems of the Copenhagen interpretation.
>> When exactly does this splitting occur? Can the split be into
>> irrational numbers of branches? A splitting is in some particular
>> basis and not in other bases. What determines the pointer basis?
>>
>> Brent
>
> --
> You received this message because you are subscribed to the Google
> Groups "Everything List" group.
> To unsubscribe from this group and stop receiving emails from it, send
> an email to everything-li...@googlegroups.com.
> To view this discussion on the web visit
> https://groups.google.com/d/msgid/everything-list/CAFxXSLTBdJpDkw_duZDuvMvArLte-3OoxJcs8-3vXjroKSti8g%40mail.gmail.com
> [1].
>
>
> Links:
> ------
> [1]
> https://groups.google.com/d/msgid/everything-list/CAFxXSLTBdJpDkw_duZDuvMvArLte-3OoxJcs8-3vXjroKSti8g%40mail.gmail.com?utm_medium=email&utm_source=footer

Brent Meeker

unread,
May 3, 2022, 1:52:37 PM5/3/22
to everyth...@googlegroups.com
Is there any evidence that is NOT from collapse?  How does it get
recorded?  Where is it?  A credible theoretical model is one that
predicts the observed result...not necessarily one that satisfies your
metaphysical prejudices.  You seem to have adopted a Platonist view of
physics.  But as Sean Carroll (a proponent of MWI) remarked, "But all
human progress has come from studying the shadows on the wall."

Brent

Bruce Kellett

unread,
May 3, 2022, 7:49:32 PM5/3/22
to Everything List
On Tue, May 3, 2022 at 10:11 PM smitra <smi...@zonnet.nl> wrote:
On 28-04-2022 07:51, Bruce Kellett wrote:
> On Thu, Apr 28, 2022 at 3:24 PM Brent Meeker <meeke...@gmail.com>
> wrote:
>
>> On 4/26/2022 5:32 PM, smitra wrote:
>>
>>> On 27-04-2022 01:37, Bruce Kellett wrote:
>> Changing the weights of the components in the superposition does not
>> change the conclusion of most observers that the actual probabilities
>> are 0.5 for each result. This is simple mathematics, and I am amazed
>> that even after all these years, and all the times I have spelled this
>> out, you still seek to deny the obvious result. Your logical and
>> mathematical skill are on a par with those of John Clark.
>>
>> It's indeed simple mathematics. You apply that to branch counting to
>> arrive at the result of equal probabilities.
>
> I have not used branch counting. Please stop accusing me of that.
>

You are considering each branch to have an equal probability when there
is no logical reason to do so, and when that's also being contradicted
by QM.

I have not introduced any concept of probability. The 2^N branches that are constructed when both outcomes are realized on each of N Bernoulli trials are all on the same basis. There is no probability involved. The branches are all equivalent by construction.

I think you are being confused by the presence of coefficients in the expansion of the original state: the a and b in

      |psi> = a|0> + b|1>

The linearity of the Schrodinger equation means that the coefficients, a and b, play no part in the construction of the 2^N possible branches; you get the same set of 2^N branches whatever the values of a and b. Think of it this way. If a = sqrt(0.9) and b = sqrt(0.1), the Born rule probability for |0> is 90%, and the Born rule probability for |1> is 10%. But, by hypothesis, both outcomes occur with certainty on each trial. There is a conflict here. You cannot rationally have a 10% probability for something that is certain to happen. This is why some people have resorted to the idea that there are in fact an infinite number of branches, both before and after the measurement. What the measurement does is partition these branches in the ratio of the Born probabilities. But this is just a suggestion. There is nothing in the Schrodinger equation, or in quantum mechanics itself, that would suggest that there are an infinite number of branches. In fact, that idea introduces a raft of problems of its own -- what is the measure over this infinity of branches? What does it mean to partition infinity in the ratio of 0.9:0.1? What is the mechanism (necessarily outside the Schrodinger equation) that achieves this?

You are concerned that a collapse introduces unknown physics outside the Schrodinger equation. You will have to be careful that your own solution does not introduce even more outrageous physics outside the Schrodinger equation. Collapse, after all, has a perfectly reasonable mechanism in terms of the flashes of relativistic GRW theory.

My conclusion from this is that Everett (and MWI) is inconsistent with the Born rule. So your idea of QM without collapse but with the Born rule, is simply incoherent. There can be no such theory that is internally consistent.

The amplitudes don't matter because the Schrodinger equation is insensitive to these amplitudes. The Born rule is simply an imposition from outside -- it is not derivable from the SE itself. The amplitudes matter only once you assume that the theory is probabilistic, then the amplitudes, through the guessed Born rule, give you a measure of these probabilities. But the SE itself, as interpreted by Everett, is deterministic, not probabilistic. There are no probabilities in the SE. So before you introduce probabilities and the Born rule, the amplitudes do not make any difference. Probabilities were introduced in order to connect quantum mechanics with the experimental evidence in one world. And probabilities make sense only in this context. Strictly, the wave function makes sense only as a way to predict and calculate probabilities. The idea of wave function realism is just metaphysics -- there is no experimental evidence for such an idea.



>>> There is nothing in QM that says that branches must count equally,
>>> and the lottery example I gave makes it clear that you can have
>>> branching with unequal probabilities in classical physics.
>
> As I have said, there is no classical analogue of an interaction in
> which all outcomes necessarily occur. So your lottery example is
> useless. There is no concept of probability involved in any of this.
>

The lottery example I gave clearly is a classical example in which all
outcomes necessarily occur.

That is a matter of interpreting what an outcome is. In normal parlance, the outcome of a lottery is the drawing of a winning ticket. There is only one draw, one winning ticket, one outcome. The other possible outcomes (which do not occur) are represented by other draws with different winning tickets. So, as in all classical cases, there is never a situation in which all possible outcomes occur.

You could take the view that not having the winning ticket is as much an outcome as winning the lottery. This is a bit contrived, but it does allow you to say that all outcomes are realized in that all numbered tickets exist, even though only one of them wins. The probability of winning is then reduced to branch counting, and the concept of an outcome is reduced to a triviality. Whereas, in the case where the winning ticket is the only outcome of relevance, the probability of winning is given solely by the number of tickets on issue. It has nothing to do with branch counting.


Your reasoning does not involve any QM at
all, you just apply it to the MWI. Your argument goes through also in
case of the lottery example, in which case it leads to an obviopusly
wrong conclusion. So, it's your reasoning that's at fault not the MWI
taken to be QM minus collapse.

My reasoning does involve QM in an essential way. A quantum state is a vector in Hilbert space that can be expanded in terms of some set of basis states. If these basis states are pointer states, stable under decoherence, and each is the eigenstate of some operator with some eigenvalue, then the set of all possible outcomes of a trial is the set of all these eigenvalues (assumed, for convenience, to be distinct). These are distinctive quantum concepts that have no classical analogues.

Bruce

John Clark

unread,
May 4, 2022, 5:51:47 AM5/4/22
to 'Brent Meeker' via Everything List
On Tue, May 3, 2022 at 1:52 PM Brent Meeker <meeke...@gmail.com> wrote:

> Is there any evidence that is NOT from collapse?  

Yes, from the 2 slit experiment.  

> How does it get recorded?  

 Data gets recorded the same way all data is recorded,  it's recorded on data recorders.

> Where is it?

I don't know, wherever you keep your data recorder.

John K Clark    See what's on my new list at  Extropolis
rda

  

John Clark

unread,
May 4, 2022, 6:27:35 AM5/4/22
to 'Brent Meeker' via Everything List
On Tue, May 3, 2022 at 7:49 PM Bruce Kellett <bhkel...@gmail.com> wrote:

> Collapse, after all, has a perfectly reasonable mechanism in terms of the flashes of relativistic GRW theory.

After the big brouhaha we had about energy conservation and how you thought it should be considered a sacred principle, it seems odd you would bring up GRW theory as an alternative to many worlds since the modification to Schrodinger's equation GRW requires violates the law of conservation of energy. And it violates the law of conservation of momentum. And it causes  Schrodinger to be nondeterministic. And it makes solving the Schrodinger equation, which was already quite difficult, even more difficult. And there is not a shred of experimental evidence that it's actually true. And all these painful contortions were performed for one reason and one reason only, to get rid of those nasty many worlds that some people find emotionally distasteful.

I say junk all those ridiculous bells and whistles and strip quantum mechanics down to its essentials.


John K Clark    See what's on my new list at  Extropolis
oxc

smitra

unread,
May 4, 2022, 2:36:39 PM5/4/22
to everyth...@googlegroups.com
A theoretical model cannot be tied to macroscopic concepts that are
known to only give an effective description of nature. It's just like
concepts in thermodynamics that can be explained in a more fundamental
way using statistical physics. No one objects to doing that on the
grounds of any practical impossibility of building molecular-scale heat
engines.

Saibal



> Brent

smitra

unread,
May 4, 2022, 3:27:32 PM5/4/22
to everyth...@googlegroups.com
If you ignore the amplitudes in the states, and that means modifying QM
into something else.

> There is no probability
> involved. The branches are all equivalent by construction.
>

What you are constructing is not the result of QM.

> I think you are being confused by the presence of coefficients in the
> expansion of the original state: the a and b in
>
> |psi> = a|0> + b|1>
>
> The linearity of the Schrodinger equation means that the coefficients,
> a and b, play no part in the construction of the 2^N possible
> branches; you get the same set of 2^N branches whatever the values of
> a and b. Think of it this way. If a = sqrt(0.9) and b = sqrt(0.1), the
> Born rule probability for |0> is 90%, and the Born rule probability
> for |1> is 10%. But, by hypothesis, both outcomes occur with certainty
> on each trial. There is a conflict here. You cannot rationally have a
> 10% probability for something that is certain to happen.

Of course you can. The lottery example shows that even in classical
physics you can imagine this happening. If a million copies of you are
made and one will win a lottery whole the rest won't then you have one
in a million chance of experiencing winning the lottery, even though
both outcomes of winning and losing will occur with certainty. One has
to distinguish between the bird's eye and frog's eye view in a setting
where there are copies of observers.



> This is why
> some people have resorted to the idea that there are in fact an
> infinite number of branches, both before and after the measurement.
> What the measurement does is partition these branches in the ratio of
> the Born probabilities. But this is just a suggestion. There is
> nothing in the Schrodinger equation, or in quantum mechanics itself,
> that would suggest that there are an infinite number of branches.

There is the Born rule. If one throws away the Born rule then one has to
specify another model that explains where it comes from. But of you just
throw away the Born rule and don't replace that woth anythong else, then
you are obviously not going to reproduce the same results.

> In
> fact, that idea introduces a raft of problems of its own -- what is
> the measure over this infinity of branches? What does it mean to
> partition infinity in the ratio of 0.9:0.1? What is the mechanism
> (necessarily outside the Schrodinger equation) that achieves this?
>

That simply means that there is as of yet no good model for QM without
the Born rule.

> You are concerned that a collapse introduces unknown physics outside
> the Schrodinger equation. You will have to be careful that your own
> solution does not introduce even more outrageous physics outside the
> Schrodinger equation. Collapse, after all, has a perfectly reasonable
> mechanism in terms of the flashes of relativistic GRW theory.
>

If objective collapse exists then one should be able to demonstrate that
in an experiment. There are as of yet no experimental results that
suggests that a collapse mechanism exists.

> My conclusion from this is that Everett (and MWI) is inconsistent with
> the Born rule. So your idea of QM without collapse but with the Born
> rule, is simply incoherent. There can be no such theory that is
> internally consistent.
>

That's based on assuming a model for the MWI that by construction is
faulty.
The Schrodinger equation keeps track of the amplitudes. That's actually
the whole point of the Schrodinger equation. It tells you how the
amplitudes assigned to states change over time.

> The Born rule is simply an imposition
> from outside -- it is not derivable from the SE itself.

The Lorentz force equation is not derivable from the Maxwell equations
either.


> The amplitudes
> matter only once you assume that the theory is probabilistic, then the
> amplitudes, through the guessed Born rule, give you a measure of these
> probabilities. But the SE itself, as interpreted by Everett, is
> deterministic, not probabilistic. There are no probabilities in the
> SE.

The multiverse is deterministic, what observers observe isn't/

> So before you introduce probabilities and the Born rule, the
> amplitudes do not make any difference. Probabilities were introduced
> in order to connect quantum mechanics with the experimental evidence
> in one world. And probabilities make sense only in this context.
> Strictly, the wave function makes sense only as a way to predict and
> calculate probabilities. The idea of wave function realism is just
> metaphysics -- there is no experimental evidence for such an idea.
>

There is no evidence for a collapse interpretation either, an collapse
requires a new physical mechanism that has yet to be observed.

>>>>> There is nothing in QM that says that branches must count
>> equally,
>>>>> and the lottery example I gave makes it clear that you can have
>>>>> branching with unequal probabilities in classical physics.
>>>
>>> As I have said, there is no classical analogue of an interaction
>> in
>>> which all outcomes necessarily occur. So your lottery example is
>>> useless. There is no concept of probability involved in any of
>> this.
>>>
>>
>> The lottery example I gave clearly is a classical example in which
>> all
>> outcomes necessarily occur.
>
> That is a matter of interpreting what an outcome is. In normal
> parlance, the outcome of a lottery is the drawing of a winning ticket.
> There is only one draw, one winning ticket, one outcome. The other
> possible outcomes (which do not occur) are represented by other draws
> with different winning tickets. So, as in all classical cases, there
> is never a situation in which all possible outcomes occur.
>

Not of we make copies f the same person and consider a lotter draw for
those copies. Then all outcomes occur and yet there is only a small
probability of winning the lottery.

> You could take the view that not having the winning ticket is as much
> an outcome as winning the lottery. This is a bit contrived, but it
> does allow you to say that all outcomes are realized in that all
> numbered tickets exist, even though only one of them wins. The
> probability of winning is then reduced to branch counting, and the
> concept of an outcome is reduced to a triviality. Whereas, in the case
> where the winning ticket is the only outcome of relevance, the
> probability of winning is given solely by the number of tickets on
> issue. It has nothing to do with branch counting.
>

This changes when you consider copies of the same observer.

>> Your reasoning does not involve any QM at
>> all, you just apply it to the MWI. Your argument goes through also
>> in
>> case of the lottery example, in which case it leads to an obviopusly
>>
>> wrong conclusion. So, it's your reasoning that's at fault not the
>> MWI
>> taken to be QM minus collapse.
>
> My reasoning does involve QM in an essential way. A quantum state is a
> vector in Hilbert space that can be expanded in terms of some set of
> basis states. If these basis states are pointer states, stable under
> decoherence, and each is the eigenstate of some operator with some
> eigenvalue, then the set of all possible outcomes of a trial is the
> set of all these eigenvalues (assumed, for convenience, to be
> distinct). These are distinctive quantum concepts that have no
> classical analogues.
>

Yes, but you can't omit the amplitudes. The reasoning you use to do that
is not based on QM and that reasoning can be applied to classical cases
too where it then also gives the wrong results.

Saibal

> Bruce
>
> --
> You received this message because you are subscribed to the Google
> Groups "Everything List" group.
> To unsubscribe from this group and stop receiving emails from it, send
> an email to everything-li...@googlegroups.com.
> To view this discussion on the web visit
> https://groups.google.com/d/msgid/everything-list/CAFxXSLTpLx_OjCRwjn2PBPeyNN3tLQydV7Qi64RNpKOmseug3g%40mail.gmail.com
> [1].
>
>
> Links:
> ------
> [1]
> https://groups.google.com/d/msgid/everything-list/CAFxXSLTpLx_OjCRwjn2PBPeyNN3tLQydV7Qi64RNpKOmseug3g%40mail.gmail.com?utm_medium=email&utm_source=footer

Brent Meeker

unread,
May 4, 2022, 4:24:15 PM5/4/22
to everyth...@googlegroups.com
But that's not "known".  It's only "known" if you assume the theoretical
model...circular reasoning.

> It's just like concepts in thermodynamics that can be explained in a
> more fundamental way using statistical physics. No one objects to
> doing that on the grounds of any practical impossibility of building
> molecular-scale heat engines.

But the consequences of thermodynamics are confirmed by observation. 
MWI puts them where they are, in principle, unobservable.

Brent

Brent Meeker

unread,
May 4, 2022, 6:04:05 PM5/4/22
to everyth...@googlegroups.com


On 5/4/2022 12:27 PM, smitra wrote:
In
fact, that idea introduces a raft of problems of its own -- what is
the measure over this infinity of branches? What does it mean to
partition infinity in the ratio of 0.9:0.1? What is the mechanism
(necessarily outside the Schrodinger equation) that achieves this?


That simply means that there is as of yet no good model for QM without the Born rule.

But there is no mechanism for the Born rule.  It is inconsistent with pure Schroedinger evolution of the wave function.  I think the problem of measures on infinity is overcome if you simply postulate a very large but finite number of branches to split.  Or why not not an continuum probability and just measure by the density around the eigenvalue...the measured values are never exact anyway.  I don't these things are wrong or show MWI is inconsistent, but I think they show it has just moved the problems it purported to solve off to some unobservable worlds, which is no better than CI.

Brent

Bruce Kellett

unread,
May 4, 2022, 7:01:58 PM5/4/22
to Everything List
On Thu, May 5, 2022 at 8:04 AM Brent Meeker <meeke...@gmail.com> wrote:
On 5/4/2022 12:27 PM, smitra wrote:
In fact, that idea introduces a raft of problems of its own -- what is
the measure over this infinity of branches? What does it mean to
partition infinity in the ratio of 0.9:0.1? What is the mechanism
(necessarily outside the Schrodinger equation) that achieves this?

That simply means that there is as of yet no good model for QM without the Born rule.

But there is no mechanism for the Born rule.  It is inconsistent with pure Schroedinger evolution of the wave function.  I think the problem of measures on infinity is overcome if you simply postulate a very large but finite number of branches to split.

The trouble if the number of branches is finite is that, given the large number of splits since the beginning of time, you will eventually run out of branches to split.
Or why not not an continuum probability and just measure by the density around the eigenvalue

How do you measure the density? You still need to impose a measure on an infinite set.
...the measured values are never exact anyway.  I don't these things are wrong or show MWI is inconsistent, but I think they show it has just moved the problems it purported to solve off to some unobservable worlds, which is no better than CI.

They show that MWI, as proposed by Everett, cannot work without such extensive modification that it is no longer the same theory. What is more, the required modifications are all ad hoc patches -- you lose any claim to rigour.

Bruce

Bruce Kellett

unread,
May 4, 2022, 7:15:52 PM5/4/22
to Everything List
On Thu, May 5, 2022 at 5:27 AM smitra <smi...@zonnet.nl> wrote:
On 04-05-2022 01:49, Bruce Kellett wrote:
>
> I have not introduced any concept of probability. The 2^N branches
> that are constructed when both outcomes are realized on each of N
> Bernoulli trials are all on the same basis.

If you ignore the amplitudes in the states, and that means modifying QM
into something else.

QM does not assume that all branches exist equally. In Everett you have already modified QM into something else.

The Schrodinger equation is insensitive to the amplitudes. You get the same set of 2^N branches from the Schrodinger equation, whatever amplitudes you have. The weights of these branches certainly depend on the amplitudes: if there are n zeros in the set of N trials, there are N-n ones. The weight of the corresponding binary string is a^n b^(N-n), but without further assumption, this plays no role in the future development of the state or in the interpretation of the binary string. If you interpret it as the probability of the string, you again have a conflict, since all binary strings are constructed on an equal basis, the natural probability for each is 2^{-N}. Because of these obvious problems, most writers on MWI interpret the coefficients as weights, and are careful to avoid calling the amplitudes probabilities. The Born rule is taken to sit alongside the theory, but it is not part of the theory because there are no probabilities in the Schrodinger equation itself.

Bruce 

Brent Meeker

unread,
May 4, 2022, 7:57:04 PM5/4/22
to everyth...@googlegroups.com


On 5/4/2022 4:01 PM, Bruce Kellett wrote:
On Thu, May 5, 2022 at 8:04 AM Brent Meeker <meeke...@gmail.com> wrote:
On 5/4/2022 12:27 PM, smitra wrote:
In fact, that idea introduces a raft of problems of its own -- what is
the measure over this infinity of branches? What does it mean to
partition infinity in the ratio of 0.9:0.1? What is the mechanism
(necessarily outside the Schrodinger equation) that achieves this?

That simply means that there is as of yet no good model for QM without the Born rule.

But there is no mechanism for the Born rule.  It is inconsistent with pure Schroedinger evolution of the wave function.  I think the problem of measures on infinity is overcome if you simply postulate a very large but finite number of branches to split.

The trouble if the number of branches is finite is that, given the large number of splits since the beginning of time, you will eventually run out of branches to split.

There's always a bigger, but still finite number.  Hilbert space already assumes continuous complex values.


Or why not not an continuum probability and just measure by the density around the eigenvalue

How do you measure the density? You still need to impose a measure on an infinite set.

The reals have natural measurable subsets which define the Lebesque measure.

https://e.math.cornell.edu/people/belk/measuretheory/LebesgueMeasure.pdf

Brent


...the measured values are never exact anyway.  I don't these things are wrong or show MWI is inconsistent, but I think they show it has just moved the problems it purported to solve off to some unobservable worlds, which is no better than CI.

They show that MWI, as proposed by Everett, cannot work without such extensive modification that it is no longer the same theory. What is more, the required modifications are all ad hoc patches -- you lose any claim to rigour.

Bruce
--
You received this message because you are subscribed to the Google Groups "Everything List" group.
To unsubscribe from this group and stop receiving emails from it, send an email to everything-li...@googlegroups.com.

Bruce Kellett

unread,
May 4, 2022, 7:57:48 PM5/4/22
to Everything List
On Thu, May 5, 2022 at 5:27 AM smitra <smi...@zonnet.nl> wrote:
On 04-05-2022 01:49, Bruce Kellett wrote:
> On Tue, May 3, 2022 at 10:11 PM smitra <smi...@zonnet.nl> wrote:
>
What you are constructing is not the result of QM.

> I think you are being confused by the presence of coefficients in the
> expansion of the original state: the a and b in
>
>       |psi> = a|0> + b|1>
>
> The linearity of the Schrodinger equation means that the coefficients,
> a and b, play no part in the construction of the 2^N possible
> branches; you get the same set of 2^N branches whatever the values of
> a and b. Think of it this way. If a = sqrt(0.9) and b = sqrt(0.1), the
> Born rule probability for |0> is 90%, and the Born rule probability
> for |1> is 10%. But, by hypothesis, both outcomes occur with certainty
> on each trial. There is a conflict here. You cannot rationally have a
> 10% probability for something that is certain to happen.

Of course you can. The lottery example shows that even in classical
physics you can imagine this happening. If  a million copies of you are
made and one will win a lottery whole the rest won't then you have one
in a million chance of experiencing winning the lottery, even though
both outcomes of winning and losing will occur with certainty.

The trouble is that classically, a million copies of you cannot be made. The issue was that if the probability of an outcome is 10%, then it does not make sense to say that that outcome will certainly happen. Putting things off into other worlds does not make the logic work. If there is a copy of you for every ticket in the lottery, then you can say with certainty that one copy of you will have the winning ticket. But what sense does it make to say that your chance of winning is then one in a million? You can't have it both ways. If winning and not winning are both regarded as legitimate outcomes, then you are not certain to win, although you are certain to have an outcome. Whatever way you spin it, the same thing cannot both be certain and have a probability of 10% (or one in a million).

Bruce

Bruce Kellett

unread,
May 4, 2022, 8:17:07 PM5/4/22
to Everything List
On Thu, May 5, 2022 at 9:57 AM Brent Meeker <meeke...@gmail.com> wrote:
On 5/4/2022 4:01 PM, Bruce Kellett wrote:
On Thu, May 5, 2022 at 8:04 AM Brent Meeker <meeke...@gmail.com> wrote:
On 5/4/2022 12:27 PM, smitra wrote:
In fact, that idea introduces a raft of problems of its own -- what is
the measure over this infinity of branches? What does it mean to
partition infinity in the ratio of 0.9:0.1? What is the mechanism
(necessarily outside the Schrodinger equation) that achieves this?

That simply means that there is as of yet no good model for QM without the Born rule.

But there is no mechanism for the Born rule.  It is inconsistent with pure Schroedinger evolution of the wave function.  I think the problem of measures on infinity is overcome if you simply postulate a very large but finite number of branches to split.

The trouble if the number of branches is finite is that, given the large number of splits since the beginning of time, you will eventually run out of branches to split.

There's always a bigger, but still finite number.  Hilbert space already assumes continuous complex values.

You cannot adjust the total number of branches as you go: you can't manufacture more branches if you run short

Or why not not an continuum probability and just measure by the density around the eigenvalue

How do you measure the density? You still need to impose a measure on an infinite set.

The reals have natural measurable subsets which define the Lebesque measure.

https://e.math.cornell.edu/people/belk/measuretheory/LebesgueMeasure.pdf


Can you put the infinite set of branches in one-to-one correspondence with the reals? Are these, in fact, equivalent sets. What is the length of a set of branches?

I think there might be problems with using the Lebesgue measure over sets of branches. You can define a Lebesgue measure over the real line because there is a natural concept of the length of an interval. There is no such natural concept of length over a set of branches.

Bruce

Brent Meeker

unread,
May 4, 2022, 10:12:38 PM5/4/22
to everyth...@googlegroups.com


On 5/4/2022 5:16 PM, Bruce Kellett wrote:
On Thu, May 5, 2022 at 9:57 AM Brent Meeker <meeke...@gmail.com> wrote:
On 5/4/2022 4:01 PM, Bruce Kellett wrote:
On Thu, May 5, 2022 at 8:04 AM Brent Meeker <meeke...@gmail.com> wrote:
On 5/4/2022 12:27 PM, smitra wrote:
In fact, that idea introduces a raft of problems of its own -- what is
the measure over this infinity of branches? What does it mean to
partition infinity in the ratio of 0.9:0.1? What is the mechanism
(necessarily outside the Schrodinger equation) that achieves this?

That simply means that there is as of yet no good model for QM without the Born rule.

But there is no mechanism for the Born rule.  It is inconsistent with pure Schroedinger evolution of the wave function.  I think the problem of measures on infinity is overcome if you simply postulate a very large but finite number of branches to split.

The trouble if the number of branches is finite is that, given the large number of splits since the beginning of time, you will eventually run out of branches to split.

There's always a bigger, but still finite number.  Hilbert space already assumes continuous complex values.

You cannot adjust the total number of branches as you go: you can't manufacture more branches if you run short

Why not?  "Being a branch" is only a matter of degree anyway.  There are a bazillion weakly decohered states every second which our instruments could not distinguish.



Or why not not an continuum probability and just measure by the density around the eigenvalue

How do you measure the density? You still need to impose a measure on an infinite set.

The reals have natural measurable subsets which define the Lebesque measure.

https://e.math.cornell.edu/people/belk/measuretheory/LebesgueMeasure.pdf


Can you put the infinite set of branches in one-to-one correspondence with the reals? Are these, in fact, equivalent sets. What is the length of a set of branches?

I think there might be problems with using the Lebesgue measure over sets of branches. You can define a Lebesgue measure over the real line because there is a natural concept of the length of an interval. There is no such natural concept of length over a set of branches.

If the branches differ by a real parameter, like the time of the radioactive decay for Schoerdinger's cat, it should work.  In general you might have to come up with something like Zurek's quantum Darwinism to provide a measure.

Brent


Bruce
--
You received this message because you are subscribed to the Google Groups "Everything List" group.
To unsubscribe from this group and stop receiving emails from it, send an email to everything-li...@googlegroups.com.

Bruce Kellett

unread,
May 4, 2022, 10:27:26 PM5/4/22
to Everything List
On Thu, May 5, 2022 at 12:12 PM Brent Meeker <meeke...@gmail.com> wrote:
On 5/4/2022 5:16 PM, Bruce Kellett wrote:
On Thu, May 5, 2022 at 9:57 AM Brent Meeker <meeke...@gmail.com> wrote:
On 5/4/2022 4:01 PM, Bruce Kellett wrote:
On Thu, May 5, 2022 at 8:04 AM Brent Meeker <meeke...@gmail.com> wrote:
On 5/4/2022 12:27 PM, smitra wrote:
In fact, that idea introduces a raft of problems of its own -- what is
the measure over this infinity of branches? What does it mean to
partition infinity in the ratio of 0.9:0.1? What is the mechanism
(necessarily outside the Schrodinger equation) that achieves this?

That simply means that there is as of yet no good model for QM without the Born rule.

But there is no mechanism for the Born rule.  It is inconsistent with pure Schroedinger evolution of the wave function.  I think the problem of measures on infinity is overcome if you simply postulate a very large but finite number of branches to split.

The trouble if the number of branches is finite is that, given the large number of splits since the beginning of time, you will eventually run out of branches to split.

There's always a bigger, but still finite number.  Hilbert space already assumes continuous complex values.

You cannot adjust the total number of branches as you go: you can't manufacture more branches if you run short

Why not?  "Being a branch" is only a matter of degree anyway.  There are a bazillion weakly decohered states every second which our instruments could not distinguish.

You make the whole concept meaningless.

Or why not not an continuum probability and just measure by the density around the eigenvalue

How do you measure the density? You still need to impose a measure on an infinite set.

The reals have natural measurable subsets which define the Lebesque measure.

https://e.math.cornell.edu/people/belk/measuretheory/LebesgueMeasure.pdf


Can you put the infinite set of branches in one-to-one correspondence with the reals? Are these, in fact, equivalent sets. What is the length of a set of branches?

I think there might be problems with using the Lebesgue measure over sets of branches. You can define a Lebesgue measure over the real line because there is a natural concept of the length of an interval. There is no such natural concept of length over a set of branches.

If the branches differ by a real parameter, like the time of the radioactive decay for Schoerdinger's cat, it should work.  In general you might have to come up with something like Zurek's quantum Darwinism to provide a measure.

The obvious point is that the branches are not generally ordered, and are not Lebesgue measurable. It is not sufficient to claim that some measure should be possible. You have to find a measure that works for all infinite sets of branches. Quantum Darwinism is not even a starter.

Bruce

smitra

unread,
May 7, 2022, 9:10:01 PM5/7/22
to everyth...@googlegroups.com
If collapse is not effective but a real effect not due to decoherence,
then there is as of yet no experimental evidence for it.

>> It's just like concepts in thermodynamics that can be explained in a
>> more fundamental way using statistical physics. No one objects to
>> doing that on the grounds of any practical impossibility of building
>> molecular-scale heat engines.
>
> But the consequences of thermodynamics are confirmed by observation. 
> MWI puts them where they are, in principle, unobservable.
>

Real collapse would have clear observational consequences. There is no
experimental evidence for collapse. A real collapse would also violate
QM, despite being part of the postulated of QM as traditionally
formulated, due to the Schrödinger equation not being universally valid.

Saibal

> Brent

smitra

unread,
May 7, 2022, 9:21:39 PM5/7/22
to everyth...@googlegroups.com
Born rule is not inconsistent with the Schrödinger equation, it just
tells you that the wavefunction gives you the probability amplitudes.
This is better than the CI, because the CI is inconsistent with the
Schrödinger equation.

The issues with branches etc. are likely just artifacts with making
hidden assumptions about branches. At the end of the day there are only
a finite number of states an observer can be in. If an observer is
modeled as an algorithm, take e.g. Star Trek's Mr. Data then it's clear
that there are only a finite number of bitstrings that can correspond to
the set of all possible things Mr. Data can be aware of.

Saibal


> Brent
>
> --
> You received this message because you are subscribed to the Google
> Groups "Everything List" group.
> To unsubscribe from this group and stop receiving emails from it, send
> an email to everything-li...@googlegroups.com.
> To view this discussion on the web visit
> https://groups.google.com/d/msgid/everything-list/24e759f9-cf72-8439-a8e7-cf3fc63cd2b7%40gmail.com
> [1].
>
>
> Links:
> ------
> [1]
> https://groups.google.com/d/msgid/everything-list/24e759f9-cf72-8439-a8e7-cf3fc63cd2b7%40gmail.com?utm_medium=email&utm_source=footer

smitra

unread,
May 7, 2022, 9:32:26 PM5/7/22
to everyth...@googlegroups.com
On 05-05-2022 01:15, Bruce Kellett wrote:
> On Thu, May 5, 2022 at 5:27 AM smitra <smi...@zonnet.nl> wrote:
>
>> On 04-05-2022 01:49, Bruce Kellett wrote:
>>>
>>> I have not introduced any concept of probability. The 2^N branches
>>> that are constructed when both outcomes are realized on each of N
>>> Bernoulli trials are all on the same basis.
>>
>> If you ignore the amplitudes in the states, and that means modifying
>> QM
>> into something else.
>
> QM does not assume that all branches exist equally. In Everett you
> have already modified QM into something else.
>
> The Schrodinger equation is insensitive to the amplitudes. You get the
> same set of 2^N branches from the Schrodinger equation, whatever
> amplitudes you have. The weights of these branches certainly depend on
> the amplitudes: if there are n zeros in the set of N trials, there are
> N-n ones. The weight of the corresponding binary string is a^n
> b^(N-n), but without further assumption, this plays no role in the
> future development of the state or in the interpretation of the binary
> string. If you interpret it as the probability of the string, you
> again have a conflict, since all binary strings are constructed on an
> equal basis, the natural probability for each is 2^{-N}.

There is no conflict whatsoever with assuming the Born rule and the
Schrodinger equation. The "construction on an equal basis" is not at all
implied by the Schrödinger equation.

> Because of
> these obvious problems, most writers on MWI interpret the coefficients
> as weights, and are careful to avoid calling the amplitudes
> probabilities. The Born rule is taken to sit alongside the theory, but
> it is not part of the theory because there are no probabilities in the
> Schrodinger equation itself.
>

There are no forces in Maxwell equations either, that's given by the
Lorentz force equation. I sits alongside the Maxwell equations.

Saibal

> Bruce
>
> --
> You received this message because you are subscribed to the Google
> Groups "Everything List" group.
> To unsubscribe from this group and stop receiving emails from it, send
> an email to everything-li...@googlegroups.com.
> To view this discussion on the web visit
> https://groups.google.com/d/msgid/everything-list/CAFxXSLR6FM1csyWpsOMVdtjCMmDdik_i7j7ka-CeO1hd7tSfaA%40mail.gmail.com
> [1].
>
>
> Links:
> ------
> [1]
> https://groups.google.com/d/msgid/everything-list/CAFxXSLR6FM1csyWpsOMVdtjCMmDdik_i7j7ka-CeO1hd7tSfaA%40mail.gmail.com?utm_medium=email&utm_source=footer

smitra

unread,
May 7, 2022, 9:40:02 PM5/7/22
to everyth...@googlegroups.com
Then assume that I'm Mr. Data and just copy the software running Mr.
Data a million times. So, this is not a findamtnel problem with the
argument.

> The issue was that if the probability of an outcome is 10%, then
> it does not make sense to say that that outcome will certainly happen.

It does make sense in a scenario where there are multiple copies if the
same observer. If Alice makes 10 copies of Bob, and one copy of Bob is
going to experience outcome A and the rest will experience outcome B,
then from Alice will see all the possible states for Bob. But from Bob's
point of view, things are different. After Bob is exposed to the result
(A or B) there are two versions of Bob, Bob<A and Bob_B, and if Bob
knows beforehand how the experiment s set up, he'll assign a probability
of 10% of going to find himself in state Bob_B after the experiment.


> Putting things off into other worlds does not make the logic work. If
> there is a copy of you for every ticket in the lottery, then you can
> say with certainty that one copy of you will have the winning ticket.
> But what sense does it make to say that your chance of winning is then
> one in a million? You can't have it both ways. If winning and not
> winning are both regarded as legitimate outcomes, then you are not
> certain to win, although you are certain to have an outcome. Whatever
> way you spin it, the same thing cannot both be certain and have a
> probability of 10% (or one in a million).
>

See above explanation.

Saibal

> Bruce
>
> --
> You received this message because you are subscribed to the Google
> Groups "Everything List" group.
> To unsubscribe from this group and stop receiving emails from it, send
> an email to everything-li...@googlegroups.com.
> To view this discussion on the web visit
> https://groups.google.com/d/msgid/everything-list/CAFxXSLQ4po5iHWyefMkk5-5AheiRTudkfkSJ2eXgfFAXX1ntTQ%40mail.gmail.com
> [1].
>
>
> Links:
> ------
> [1]
> https://groups.google.com/d/msgid/everything-list/CAFxXSLQ4po5iHWyefMkk5-5AheiRTudkfkSJ2eXgfFAXX1ntTQ%40mail.gmail.com?utm_medium=email&utm_source=footer

Bruce Kellett

unread,
May 7, 2022, 11:56:26 PM5/7/22
to Everything List
On Sun, May 8, 2022 at 11:40 AM smitra <smi...@zonnet.nl> wrote:
On 05-05-2022 01:57, Bruce Kellett wrote:
> On Thu, May 5, 2022 at 5:27 AM smitra <smi...@zonnet.nl> wrote:
>>
>> Of course you can. The lottery example shows that even in classical
>> physics you can imagine this happening. If  a million copies of you are
>> made and one will win a lottery whole the rest won't then you have one
>> in a million chance of experiencing winning the lottery, even though
>> both outcomes of winning and losing will occur with certainty.
>
> The trouble is that classically, a million copies of you cannot be
> made.

Then assume that I'm Mr. Data and just copy the software running Mr.
Data a million times. So, this is not a findamtnel problem with the
argument.

That technology does not currently exist. And one might reasonably doubt that it will ever exist....


> The issue was that if the probability of an outcome is 10%, then
> it does not make sense to say that that outcome will certainly happen.

It does make sense in a scenario where there are multiple copies if the
same observer. If Alice makes 10 copies of Bob, and one copy of Bob is
going to experience outcome A and the rest will experience outcome B,
then from Alice will see all the possible states for Bob. But from Bob's
point of view, things are different. After Bob is exposed to the result
(A or B) there are two versions of Bob, Bob<A and Bob_B, and if Bob
knows beforehand how the experiment s set up, he'll assign a probability
of 10% of going to find himself in state Bob_B after the experiment.

I think this boils down to the first person:third person confusion that Bruno often refers to.
From the third person perspective, the outcome is certain. But from the first person perspective of each of the copies, the outcome is not certain.

Consider the following simple situation. You have a bag containing ten balls, nine of which are red and one is black. If there are ten copies of Bob, for example, and each copy draws a ball from the bag, without replacement. Then it is certain (100% probability) that the black ball will be drawn. But the probability that any particular copy of Bob drew the black ball is only 10%. (They draw the balls without knowing the results of other draws). The probability that 'Bob' (including all copies, presumed identical) will have the black ball is still 100%. That is the 3p perspective. For each copy, however, their 1p perspective is that the probability that their ball is black is only 10%. The problem arises if you attempt to impose the 1p perspective on the 3p view. It cannot be the case that a particular copy of Bob is both certain to draw black and has only a 10% chance of drawing black. To consider all copies as equally identified as 'Bob' is the 3p view, and that is the view that is relevant for the Everett interpretation of an experiment -- there is nothing in the SE that identifies one particular observer (there is no 1p view), so Everett is incompatible with the Born rule (which is a 1p view).

Bruce

Bruce Kellett

unread,
May 7, 2022, 11:58:53 PM5/7/22
to Everything List
On Sun, May 8, 2022 at 11:32 AM smitra <smi...@zonnet.nl> wrote:
On 05-05-2022 01:15, Bruce Kellett wrote:
> On Thu, May 5, 2022 at 5:27 AM smitra <smi...@zonnet.nl> wrote:
>
>> On 04-05-2022 01:49, Bruce Kellett wrote:
>>>
>>> I have not introduced any concept of probability. The 2^N branches
>>> that are constructed when both outcomes are realized on each of N
>>> Bernoulli trials are all on the same basis.
>>
>> If you ignore the amplitudes in the states, and that means modifying
>> QM into something else.
>
> QM does not assume that all branches exist equally. In Everett you
> have already modified QM into something else.
>
> The Schrodinger equation is insensitive to the amplitudes. You get the
> same set of 2^N branches from the Schrodinger equation, whatever
> amplitudes you have. The weights of these branches certainly depend on
> the amplitudes: if there are n zeros in the set of N trials, there are
> N-n ones. The weight of the corresponding binary string is a^n
> b^(N-n), but without further assumption, this plays no role in the
> future development of the state or in the interpretation of the binary
> string. If you interpret it as the probability of the string, you
> again have a conflict, since all binary strings are constructed on an
> equal basis, the natural probability for each is 2^{-N}.

There is no conflict whatsoever with assuming the Born rule and the
Schrodinger equation. The "construction on an equal basis" is not at all
implied by the Schrödinger equation.

It is when you take the SE to imply that all possible outcomes exist on each trial. That gives all outcomes equal status.

Bruce

Brent Meeker

unread,
May 8, 2022, 12:03:31 AM5/8/22
to everyth...@googlegroups.com


On 5/7/2022 6:21 PM, smitra wrote:
On 05-05-2022 00:04, Brent Meeker wrote:
On 5/4/2022 12:27 PM, smitra wrote:

In
fact, that idea introduces a raft of problems of its own -- what
is
the measure over this infinity of branches? What does it mean to
partition infinity in the ratio of 0.9:0.1? What is the mechanism
(necessarily outside the Schrodinger equation) that achieves this?

That simply means that there is as of yet no good model for QM
without the Born rule.

But there is no mechanism for the Born rule.  It is inconsistent with
pure Schroedinger evolution of the wave function.  I think the problem
of measures on infinity is overcome if you simply postulate a very
large but finite number of branches to split.  Or why not not an
continuum probability and just measure by the density around the
eigenvalue...the measured values are never exact anyway.  I don't
these things are wrong or show MWI is inconsistent, but I think they
show it has just moved the problems it purported to solve off to some
unobservable worlds, which is no better than CI.


Born rule is not inconsistent with the Schrödinger equation, it just tells you that the wavefunction gives you the probability amplitudes. This is better than the CI, because the CI is inconsistent with the Schrödinger equation.

Because??  It takes one more step and says "probability means something happens and other things don't."  It's not called the "Copenhagen Equation".  It's called the "Copenhagen Interpretation", i.e. how to interpret the Schroedinger equation and so it is consistent with it.



The issues with branches etc. are likely just artifacts with making hidden assumptions about branches. At the end of the day there are only a finite number of states an observer can be in. If an observer is modeled as an algorithm, take e.g. Star Trek's Mr. Data then it's clear that there are only a finite number of bitstrings that can correspond to the set of all possible things Mr. Data can be aware of.

But different Mr. Data's and different instruments can have different number of states.  So what you're suggesting is QBism.

Brent

Bruce Kellett

unread,
May 8, 2022, 12:04:45 AM5/8/22
to Everything List
On Sun, May 8, 2022 at 11:21 AM smitra <smi...@zonnet.nl> wrote:

The issues with branches etc. are likely just artifacts with making
hidden assumptions about branches. At the end of the day there are only
a finite number of states an observer can be in. If an observer is
modeled as an algorithm, take e.g. Star Trek's Mr. Data then it's clear
that there are only a finite number of bitstrings that can correspond to
the set of all possible things Mr. Data can be aware of.

Everett is supposed to be QM without observers. So the number of things that Mr Data can possibly be aware of is irrelevant. According to the SE, all branches are equivalent. All else flows from this -- there are no further "hidden assumptions about branches".

Brent Meeker

unread,
May 8, 2022, 12:24:58 AM5/8/22
to everyth...@googlegroups.com
I don't think I agree that there is any such 3p view.  There's a 3p calculation, using MWI, in which ten different "Bob" are predicted.  But no third party ever sees these ten Bobs.

When you start to rely on subjective perspectives I think you've already violated the spirit of MWI which was proposed to apply to simple instrument records as well as consciousness.  Decoherence is such an instrument that is implicit in the environment.

Brent

Bruce Kellett

unread,
May 8, 2022, 12:58:30 AM5/8/22
to Everything List
OK, no 'person' sees ten copies of Bob or ten outcomes. But it is common to use a 'super-observer' notion for this. -- the 3p calculation. The 3p view is then the objective 'view from outside'.

When you start to rely on subjective perspectives I think you've already violated the spirit of MWI which was proposed to apply to simple instrument records as well as consciousness.  Decoherence is such an instrument that is implicit in the environment.

I think that is exactly right. I introduced 1p and 3p views in an attempt to come to terms with Saibal's presentation, but strictly everything should be done by instruments -- no persons involved. 

Bruce

smitra

unread,
May 8, 2022, 4:31:26 PM5/8/22
to everyth...@googlegroups.com
I agree here, except that the wavefunction will (in general) assign
different amplitudes to different states of observers. Therefore there
is problem with the Born rule assigning different probabilities to the
observer being in different states.

Saibal

> Bruce
>
> --
> You received this message because you are subscribed to the Google
> Groups "Everything List" group.
> To unsubscribe from this group and stop receiving emails from it, send
> an email to everything-li...@googlegroups.com.
> To view this discussion on the web visit
> https://groups.google.com/d/msgid/everything-list/CAFxXSLSj-5%2BTxBEPFkYByU%2BbEOHPELDG2NGquM%3Dqqtq3%2BXqLnw%40mail.gmail.com
> [1].
>
>
> Links:
> ------
> [1]
> https://groups.google.com/d/msgid/everything-list/CAFxXSLSj-5%2BTxBEPFkYByU%2BbEOHPELDG2NGquM%3Dqqtq3%2BXqLnw%40mail.gmail.com?utm_medium=email&utm_source=footer

smitra

unread,
May 8, 2022, 4:37:56 PM5/8/22
to everyth...@googlegroups.com
All outcomes can exist without these being equally likely. One can make
models based on more branches for certain outcomes, but these are just
models that may not be correct. What matters is that such models can be
formulated in a mathematically consistent way, which demonstrates that
there is n o contradiction. The physical plausibility of such models is
another issue.

Saibal

> Bruce
>
> --
> You received this message because you are subscribed to the Google
> Groups "Everything List" group.
> To unsubscribe from this group and stop receiving emails from it, send
> an email to everything-li...@googlegroups.com.
> To view this discussion on the web visit
> https://groups.google.com/d/msgid/everything-list/CAFxXSLSMV1OBH4Zm4jO1-rY82prqxANc4ZZcOHppOgJ1p6Z5zQ%40mail.gmail.com
> [1].
>
>
> Links:
> ------
> [1]
> https://groups.google.com/d/msgid/everything-list/CAFxXSLSMV1OBH4Zm4jO1-rY82prqxANc4ZZcOHppOgJ1p6Z5zQ%40mail.gmail.com?utm_medium=email&utm_source=footer

smitra

unread,
May 8, 2022, 4:50:30 PM5/8/22
to everyth...@googlegroups.com
> i.e. how to _INTERPRET_ the Schroedinger equation and so it is
> consistent with it.
>
It's called an interpretation just like the MWI, but these are actually
different theories that make different predictions, albeit in a domain
that cannot easily be accessed experimentally.

That the CI is inconsistent with the Schrödinger equation is easy to
see. If the Schrödinger is valid, then the state of a system evolves in
a unitary way. But after a real collapse the state changes in a
non-unitary way. If we consider measuring the z-component of a spin
polarized in the x-direction using a Stern-Gerlach apparatus, then the
entire system of the spin the experimental set-up, the observer and
local environment consists of particles that should evolve according to
the Schrödinger equation. If the measurement takes one minute, then the
initial state of a patch of one light-minute diameter around the
location of the experiment maps to a final state of that patch in a
unitary way. But CI says that this does not happen because the internal
observer in the system performed a measurement that causes the state of
the system to collapse.


>> The issues with branches etc. are likely just artifacts with making
>> hidden assumptions about branches. At the end of the day there are
>> only a finite number of states an observer can be in. If an observer
>> is modeled as an algorithm, take e.g. Star Trek's Mr. Data then it's
>> clear that there are only a finite number of bitstrings that can
>> correspond to the set of all possible things Mr. Data can be aware
>> of.
>
> But different Mr. Data's and different instruments can have different
> number of states. So what you're suggesting is QBism.
>

It may fall under QBism, the question is if this is going to cause
problems that cannot be resolved well.

Saibal

> Brent
>
> --
> You received this message because you are subscribed to the Google
> Groups "Everything List" group.
> To unsubscribe from this group and stop receiving emails from it, send
> an email to everything-li...@googlegroups.com.
> To view this discussion on the web visit
> https://groups.google.com/d/msgid/everything-list/cdb187f5-6770-7216-a03f-75fca3b0c4c7%40gmail.com
> [1].
>
>
> Links:
> ------
> [1]
> https://groups.google.com/d/msgid/everything-list/cdb187f5-6770-7216-a03f-75fca3b0c4c7%40gmail.com?utm_medium=email&utm_source=footer

smitra

unread,
May 8, 2022, 4:52:58 PM5/8/22
to everyth...@googlegroups.com
Yes, but I'm not a big fan of "sticking to scripture". What matters for
me is that collapse is inconsistent with the SE, therefore we should
consider QM without collapse and see how to best to move forward on that
basis.

Saibal


> --
> You received this message because you are subscribed to the Google
> Groups "Everything List" group.
> To unsubscribe from this group and stop receiving emails from it, send
> an email to everything-li...@googlegroups.com.
> To view this discussion on the web visit
> https://groups.google.com/d/msgid/everything-list/CAFxXSLQbZrov9a6Xs5mrr0o%3DkFSmeSHKdLG%3DbwgkOF6er43RKQ%40mail.gmail.com
> [1].
>
>
> Links:
> ------
> [1]
> https://groups.google.com/d/msgid/everything-list/CAFxXSLQbZrov9a6Xs5mrr0o%3DkFSmeSHKdLG%3DbwgkOF6er43RKQ%40mail.gmail.com?utm_medium=email&utm_source=footer

Bruce Kellett

unread,
May 8, 2022, 6:34:34 PM5/8/22
to Everything List
On Mon, May 9, 2022 at 6:52 AM smitra <smi...@zonnet.nl> wrote:
On 08-05-2022 06:04, Bruce Kellett wrote:
> On Sun, May 8, 2022 at 11:21 AM smitra <smi...@zonnet.nl> wrote:
>
>> The issues with branches etc. are likely just artifacts with making
>> hidden assumptions about branches. At the end of the day there are
>> only
>> a finite number of states an observer can be in. If an observer is
>> modeled as an algorithm, take e.g. Star Trek's Mr. Data then it's
>> clear
>> that there are only a finite number of bitstrings that can
>> correspond to
>> the set of all possible things Mr. Data can be aware of.
>
> Everett is supposed to be QM without observers. So the number of
> things that Mr Data can possibly be aware of is irrelevant. According
> to the SE, all branches are equivalent. All else flows from this --
> there are no further "hidden assumptions about branches".
>

Yes, but I'm not a big fan of "sticking to scripture". What matters for
me is that collapse is inconsistent with the SE, therefore we should
consider QM without collapse and see how to best to move forward on that
basis.

That still treats the SE as indubitally true. No theory in physics is 'indubitably true'.

The Everett program is to say that the SE is all that there is -- it explains everything. That is clearly false (no Born rule in the SE), so it might be wise to doubt the universal application of the SE.

Bruce

Bruce Kellett

unread,
May 8, 2022, 6:42:15 PM5/8/22
to Everything List
On Mon, May 9, 2022 at 6:37 AM smitra <smi...@zonnet.nl> wrote:
On 08-05-2022 05:58, Bruce Kellett wrote:

> It is when you take the SE to imply that all possible outcomes exist
> on each trial. That gives all outcomes equal status.

All outcomes can exist without these being equally likely. One can make
models based on more branches for certain outcomes, but these are just
models that may not be correct.

Such models are certainly inconsistent with the SE. So if your concern is that the SE does not contain provision for a collapse, then you should doubt other theories that violate the SE. You can't have it both ways: you can't reject collapse models because they violate the SE and then embrace other models that also violate the SE. Either the SE is universally correct, or it is not.
What matters is that such models can be
formulated in a mathematically consistent way, which demonstrates that
there is n o contradiction. The physical plausibility of such models is
another issue.

This has been discussed. To allow for real number probabilities, the number of branches on each split must be infinite. The measure problem for infinite numbers of branches has not been solved. It is unlikely that any consistent measure over infinite numbers of branches can be defined. So this idea is probably a non-starter. At least other models have a reasonable chance of success.

Bruce

Brent Meeker

unread,
May 8, 2022, 7:00:19 PM5/8/22
to everyth...@googlegroups.com
Which is only a problem if one insists that the Schroedinger equation is
the whole of the theory and it is ontic.  CI denies the first and says
that measurements are projection operators because a measurements is
necessarily a classical-like result.  QBism says the whole theory is
epistemic.

> If we consider measuring the z-component of a spin polarized in the
> x-direction using a Stern-Gerlach apparatus, then the entire system
> of  the spin the experimental set-up, the observer and local
> environment consists of particles that should evolve according to the
> Schrödinger equation.

"Should"?

> If the measurement takes one minute, then the initial state of a patch
> of one light-minute diameter around the location of the experiment
> maps to a final state of that patch in a unitary way.

You seem to overlook that this one-light minute sphere also had incoming
particles and radiation which could not be accounted for the
Schroedinger equation.

> But CI says that this does not happen because the internal observer in
> the system performed a measurement that causes the state of the system
> to collapse.

Yes, that's a problem although CI+decoherence doesn't depend on an
observer.  The effect of the incoming radiation is also a problem. But
MWI doesn't solve the problem, it just assumes that the correlations are
created which have the same effect as collapse as far as the instruments
and observers are concerned.  Decoherence goes part way to solving the
problem by quantifying how the "collapse" occurs statistically in time.

Brent

Brent Meeker

unread,
May 8, 2022, 8:17:44 PM5/8/22
to everyth...@googlegroups.com


On 5/8/2022 3:42 PM, Bruce Kellett wrote:
On Mon, May 9, 2022 at 6:37 AM smitra <smi...@zonnet.nl> wrote:
On 08-05-2022 05:58, Bruce Kellett wrote:

> It is when you take the SE to imply that all possible outcomes exist
> on each trial. That gives all outcomes equal status.

All outcomes can exist without these being equally likely. One can make
models based on more branches for certain outcomes, but these are just
models that may not be correct.

Such models are certainly inconsistent with the SE. So if your concern is that the SE does not contain provision for a collapse, then you should doubt other theories that violate the SE. You can't have it both ways: you can't reject collapse models because they violate the SE and then embrace other models that also violate the SE. Either the SE is universally correct, or it is not.
What matters is that such models can be
formulated in a mathematically consistent way, which demonstrates that
there is n o contradiction. The physical plausibility of such models is
another issue.

This has been discussed. To allow for real number probabilities, the number of branches on each split must be infinite.

I don't think that's a problem.  The number of information bits within a Hubble sphere is something like the area in Planck units, which already implies the continuum is a just a convenient approximation.  If the area is N then something order 1/N would be the smallest non-zero probability.  Also there would be a cutoff for the off-diagonal terms of the density matrix.  Once all the off-diagonal terms are zero then it's like a mixed matrix and one could say that one of the diagonal terms has "happened".

Brent

Bruce Kellett

unread,
May 8, 2022, 8:25:13 PM5/8/22
to Everything List
On Mon, May 9, 2022 at 10:17 AM Brent Meeker <meeke...@gmail.com> wrote:
On 5/8/2022 3:42 PM, Bruce Kellett wrote:
On Mon, May 9, 2022 at 6:37 AM smitra <smi...@zonnet.nl> wrote:
On 08-05-2022 05:58, Bruce Kellett wrote:

> It is when you take the SE to imply that all possible outcomes exist
> on each trial. That gives all outcomes equal status.

All outcomes can exist without these being equally likely. One can make
models based on more branches for certain outcomes, but these are just
models that may not be correct.

Such models are certainly inconsistent with the SE. So if your concern is that the SE does not contain provision for a collapse, then you should doubt other theories that violate the SE. You can't have it both ways: you can't reject collapse models because they violate the SE and then embrace other models that also violate the SE. Either the SE is universally correct, or it is not.
What matters is that such models can be
formulated in a mathematically consistent way, which demonstrates that
there is n o contradiction. The physical plausibility of such models is
another issue.

This has been discussed. To allow for real number probabilities, the number of branches on each split must be infinite.

I don't think that's a problem.  The number of information bits within a Hubble sphere is something like the area in Planck units, which already implies the continuum is a just a convenient approximation.  If the area is N then something order 1/N would be the smallest non-zero probability.  Also there would be a cutoff for the off-diagonal terms of the density matrix.  Once all the off-diagonal terms are zero then it's like a mixed matrix and one could say that one of the diagonal terms has "happened".

As I have pointed out before, a finite number of branches does not work because after a certain finite number of splits, one would run out of branches to partition in anything like the way appropriate for the related probabilities. One cannot go adding more branches at that stage without rendering the whole concept meaningless. Keeping things finite has its attractions, but it does not work in this case.

Bruce

Brent Meeker

unread,
May 8, 2022, 8:32:04 PM5/8/22
to everyth...@googlegroups.com
I think it depends on how you count splits.  If the number of dof within a Hubble volume is finite, then the number of splits doesn't grow exponentially.  They get cut off when their probability becomes too small.

Brent

Bruce Kellett

unread,
May 8, 2022, 8:39:22 PM5/8/22
to Everything List
You are back to your notion of a smallest possible probability. That also runs into problems if you run a long sequence of events where one outcome has a very small probability on each trial. Try tossing a coin N times. The probability of a sequence of N heads is 1/N. What happens when this gets smaller than the smallest allowed probability? Is the next toss somehow forbidden to give head again? You are making the whole notion of probability problematic.

Bruce

Brent Meeker

unread,
May 8, 2022, 10:47:11 PM5/8/22
to everyth...@googlegroups.com
Yes, I can see a concern.  But my back-of-the envelope estimate is that the Hubble volume has the information content of ~10^96 bits.  So it would very hard experimentally to flip enough coins to test that limit.  However it would imply that you couldn't create a pseudo-random number generator that could produce random numbers with that many bits.  That would raise the question of how would you tell?  The sequence of numbers of a good pseudo-random number generator look random until you test high order correlations.

Brent

Bruce Kellett

unread,
May 8, 2022, 11:52:13 PM5/8/22
to Everything List
I don't think that the limited number of bits of information in the Hubble volume is much of a concern. I suspect that if the number of branches is finite, and there is a limit to how small a probability can be, then everything must be discrete -- space and time along with everything else. Or else you get a Zeno effect with radioactive decay. For a long-lived isotope, the probability of decay in a small time interval can be made as small as you want by taking a small enough time interval. Whether this is measurable is not really the issue. If there is a lower limit on probability, then decays are probably impossible without discrete time and space as well.

Bruce

John Clark

unread,
May 9, 2022, 6:16:04 AM5/9/22
to 'Brent Meeker' via Everything List
On Sun, May 8, 2022 at 6:34 PM Bruce Kellett <bhkel...@gmail.com> wrote:

> The Everett program is to say that the SE is all that there is -- it explains everything. 

No! The Everett program says the only assumption Quantum Mechanics needs is that the Schrodinger Equation means what it says, and nobody in their right mind thinks Quantum Mechanics as it currently exists, or the Schrodinger Equation, describes gravity.

> That is clearly false (no Born rule in the SE), 

The Born Rule is not an assumption nor is it a theory, it is an experimentally derived fact, a fact that EVERY quantum interpretation makes use of.  

> so it might be wise to doubt the universal application of the SE.

Without Schrodinger's Equation there could be no quantum wave function, and without the quantum wave function there could be no Born Rule, and without the Born Rule there could be no Quantum Mechanics. And Quantum Mechanics is the most universally applicable idea in all of science, it works on everything except gravity, so it might be wise to keep it around.

John K Clark    See what's on my new list at  Extropolis
ebk


John Clark

unread,
May 9, 2022, 6:37:13 AM5/9/22
to 'Brent Meeker' via Everything List
On Sun, May 8, 2022 at 7:00 PM Brent Meeker <meeke...@gmail.com> wrote:

On 5/8/2022 1:50 PM, smitra wrote:

>> That the CI is inconsistent with the Schrödinger equation is easy to
>> see. If the Schrödinger is valid, then the state of a system evolves
>> in a unitary way. But after a real collapse the state changes in a
>> non-unitary way.

> Which is only a problem if one insists that the Schroedinger equation is
the whole of the theory and it is ontic.  CI denies the first and says
that measurements are projection operators because a measurements is
necessarily a classical-like result.  QBism says the whole theory is
epistemic.


And all of that is fundamentally the same as "shut up and calculate ", they're just dressed up in slightly different philosophical bafflegab.  

>> If the measurement takes one minute, then the initial state of a patch
>> of one light-minute diameter around the location of the experiment
>> maps to a final state of that patch in a unitary way.

> You seem to overlook that this one-light minute sphere also had incoming particles and radiation which could not be accounted for the Schroedinger equation.

If Everett is right then when an electron makes an up/down decision it makes no difference if you think of it as the entire universe  instantly splits or as the split expanding outward at the speed of light, either way something that happens on the surface of that expanding sphere can have no effect on its center because no signal can travel faster than light.

John K Clark    See what's on my new list at  Extropolis
lft


Brent Meeker

unread,
May 9, 2022, 2:50:17 PM5/9/22
to everyth...@googlegroups.com
We keep Newtonian mechanics around too.  That doesn't mean nothing better can be found.  Schroedinger's equation isn't even the only form of QM.  There are also transactional and path integral forms and epistemic forms like QBism.  And there are also slightly different theories like Adler's emergent QM and explicit collapse theories like GRW.  All of which are the same theory at the level of "might be wise to keep around."

Brent


John K Clark    See what's on my new list at  Extropolis
ebk


--
You received this message because you are subscribed to the Google Groups "Everything List" group.
To unsubscribe from this group and stop receiving emails from it, send an email to everything-li...@googlegroups.com.

Brent Meeker

unread,
May 9, 2022, 3:18:19 PM5/9/22
to everyth...@googlegroups.com


On 5/9/2022 3:36 AM, John Clark wrote:
On Sun, May 8, 2022 at 7:00 PM Brent Meeker <meeke...@gmail.com> wrote:

On 5/8/2022 1:50 PM, smitra wrote:

>> That the CI is inconsistent with the Schrödinger equation is easy to
>> see. If the Schrödinger is valid, then the state of a system evolves
>> in a unitary way. But after a real collapse the state changes in a
>> non-unitary way.

> Which is only a problem if one insists that the Schroedinger equation is
the whole of the theory and it is ontic.  CI denies the first and says
that measurements are projection operators because a measurements is
necessarily a classical-like result.  QBism says the whole theory is
epistemic.


And all of that is fundamentally the same as "shut up and calculate ", they're just dressed up in slightly different philosophical bafflegab. 

They're not "dressed up", they are perfectly explicit in their interpretation and ontology.  Only the Priesthood of Everett resorts to snide denigration.  And their ontology does not need an continuum infinity of universes to "explain" probability.



>> If the measurement takes one minute, then the initial state of a patch
>> of one light-minute diameter around the location of the experiment
>> maps to a final state of that patch in a unitary way.

> You seem to overlook that this one-light minute sphere also had incoming particles and radiation which could not be accounted for the Schroedinger equation.

If Everett is right then when an electron makes an up/down decision it makes no difference if you think of it as the entire universe  instantly splits or as the split expanding outward at the speed of light, either way something that happens on the surface of that expanding sphere can have no effect on its center because no signal can travel faster than light.

An electron makes an up/down decision??  That's a new interpretation!  You miss the point that parts of the universe not accounted for in your SE are acting on the instrument which is interacting with the electron as it's "making a decision".  It's not the one-light-minute sphere expanding after the measurement event, it's the one-light-minute sphere contracting onto the measurement event before it.  How does the SE account for it?

Brent


John K Clark    See what's on my new list at  Extropolis
lft


--
You received this message because you are subscribed to the Google Groups "Everything List" group.
To unsubscribe from this group and stop receiving emails from it, send an email to everything-li...@googlegroups.com.

John Clark

unread,
May 9, 2022, 4:10:10 PM5/9/22
to 'Brent Meeker' via Everything List
On Mon, May 9, 2022 at 3:18 PM Brent Meeker <meeke...@gmail.com> wrote:

>> And all of that is fundamentally the same as "shut up and calculate ", they're just dressed up in slightly different philosophical bafflegab. 

> They're not "dressed up", they are perfectly explicit in their interpretation and ontology. 

Not just explicit but "perfectly explicit"?! I don't think so. I think Copenhagen and QBism are not quantum interpretations at all, they're just thinly disguised "shut up and calculate"; Pilot Wave and Many Worlds are genuine interpretations, both are deterministic but pilot wave is not local and Many Worlds is not realistic. Explicit collapse theories like GRW are not quantum interpretations at all but propose a new theory that would replace quantum mechanics, a theory that is much harder to use than quantum mechanics, is nondeterministic, has no experimental evidence in its favor, nobody knows how to make it compatible with special relativity, and GRW violates conservation of energy that some on this list think is sacred holy writ.  


>> If Everett is right then when an electron makes an up/down decision it makes no difference if you think of it as the entire universe  instantly splits or as the split expanding outward at the speed of light, either way something that happens on the surface of that expanding sphere can have no effect on its center because no signal can travel faster than light.
 
> An electron makes an up/down decision??  That's a new interpretation!  You miss the point that [...]

I'm quite sure you are not missing my point, but you were certainly trying very hard to pretend that you are.
> parts of the universe not accounted for in your SE are acting on the instrument which is interacting with the electron as it's "making a decision". 

You're talking about non-locality such as in pilot wave theory, a particle right here can instantly make a profound change to another particle on the other side of the universe without affecting anything in between. That sounds a little bit too much like magic for my taste, I agree with Isaac Newton when he said:   


"That one body may act upon another at a distance through a vacuum, without the mediation of anything else, by and through which their action and force may be conveyed from one to another, is to me so great an absurdity that I believe no man who has in philosophical matters a competent faculty of thinking can ever fall into it "
 
> It's not the one-light-minute sphere expanding after the measurement event, it's the one-light-minute sphere contracting onto the measurement event before it.  How does the SE account for it?

You tell me, you're the one pushing this cockamamie idea.
 
John K Clark    See what's on my new list at  Extropolis
ev7

smitra

unread,
May 10, 2022, 11:41:15 PM5/10/22
to everyth...@googlegroups.com
Yes, but this mans that CI is not compatible with QM as a fundamental
theory. You can't have a fundamental Schrödinger equation and then have
it not apply to some special cases.

>> If we consider measuring the z-component of a spin polarized in the
>> x-direction using a Stern-Gerlach apparatus, then the entire system
>> of  the spin the experimental set-up, the observer and local
>> environment consists of particles that should evolve according to the
>> Schrödinger equation.
>
> "Should"?
>

If MWI is correct.

>> If the measurement takes one minute, then the initial state of a patch
>> of one light-minute diameter around the location of the experiment
>> maps to a final state of that patch in a unitary way.
>
> You seem to overlook that this one-light minute sphere also had
> incoming particles and radiation which could not be accounted for the
> Schroedinger equation.
>

Yes, so one can imagine a shield keeping particles from outside that
region from interacting with particles inside the region. Weakly
interacting particles like neutrinos can enter, but they don't interact
with what's inside the interior region. So, the state of the universe
factors into a part for the inside and outside regions (where the
outside region also incudes weakly interacting particles that have moved
inside). Both parts evolve in a unitary way.

>> But CI says that this does not happen because the internal observer in
>> the system performed a measurement that causes the state of the system
>> to collapse.
>
> Yes, that's a problem although CI+decoherence doesn't depend on an
> observer.  The effect of the incoming radiation is also a problem. But
> MWI doesn't solve the problem, it just assumes that the correlations
> are created which have the same effect as collapse as far as the
> instruments and observers are concerned.  Decoherence goes part way to
> solving the problem by quantifying how the "collapse" occurs
> statistically in time.
>

Yes, one needs to consider correlations between the states of the
measurement devices and the measured systems.

Saibal

smitra

unread,
May 10, 2022, 11:51:11 PM5/10/22
to everyth...@googlegroups.com
As Brent has also pointed out, there amount of information in the
visible universe is finite. But one can also consider observers and then
each observer has a some finite memory so there are only a finite number
of branches the observer can distinguish between.

Saibal

> Bruce
>
> --
> You received this message because you are subscribed to the Google
> Groups "Everything List" group.
> To unsubscribe from this group and stop receiving emails from it, send
> an email to everything-li...@googlegroups.com.
> To view this discussion on the web visit
> https://groups.google.com/d/msgid/everything-list/CAFxXSLTbW-T8BfRjw7_Kyf1Vw%3DLVPaVDs6iaPp4hMw97DJMEJw%40mail.gmail.com
> [1].
>
>
> Links:
> ------
> [1]
> https://groups.google.com/d/msgid/everything-list/CAFxXSLTbW-T8BfRjw7_Kyf1Vw%3DLVPaVDs6iaPp4hMw97DJMEJw%40mail.gmail.com?utm_medium=email&utm_source=footer

smitra

unread,
May 10, 2022, 11:56:13 PM5/10/22
to everyth...@googlegroups.com
There is no good reason to doubt the SE without any experimental hints
that it breaks down, or any good theoretical reasons why it is likely to
break down in some regime.

Saibal

> Bruce
>
> --
> You received this message because you are subscribed to the Google
> Groups "Everything List" group.
> To unsubscribe from this group and stop receiving emails from it, send
> an email to everything-li...@googlegroups.com.
> To view this discussion on the web visit
> https://groups.google.com/d/msgid/everything-list/CAFxXSLRG_Qpwy3294cyU2BT1M9KRgO5mFkj%3DBDg4eo4LbdYuaw%40mail.gmail.com
> [1].
>
>
> Links:
> ------
> [1]
> https://groups.google.com/d/msgid/everything-list/CAFxXSLRG_Qpwy3294cyU2BT1M9KRgO5mFkj%3DBDg4eo4LbdYuaw%40mail.gmail.com?utm_medium=email&utm_source=footer

Bruce Kellett

unread,
May 11, 2022, 12:01:43 AM5/11/22
to Everything List
On Wed, May 11, 2022 at 1:51 PM smitra <smi...@zonnet.nl> wrote:
On 09-05-2022 00:42, Bruce Kellett wrote:
>
> Such models are certainly inconsistent with the SE. So if your concern
> is that the SE does not contain provision for a collapse, then you
> should doubt other theories that violate the SE. You can't have it
> both ways: you can't reject collapse models because they violate the
> SE and then embrace other models that also violate the SE. Either the
> SE is universally correct, or it is not.
>
>> What matters is that such models can be
>> formulated in a mathematically consistent way, which demonstrates that
>> there is n o contradiction. The physical plausibility of such models
>> is another issue.
>
> This has been discussed. To allow for real number probabilities, the
> number of branches on each split must be infinite. The measure problem
> for infinite numbers of branches has not been solved. It is unlikely
> that any consistent measure over infinite numbers of branches can be
> defined. So this idea is probably a non-starter. At least other models
> have a reasonable chance of success.
>

As Brent has also pointed out, there amount of information in the
visible universe is finite.

That does not limit the number of branches. A finite universe does not limit the number of points in a line.


But one can also consider observers and then
each observer has a some finite memory so there are only a finite number
of branches the observer can distinguish between.

That does not follow.

Bruce

Bruce Kellett

unread,
May 11, 2022, 12:06:24 AM5/11/22
to Everything List
On Wed, May 11, 2022 at 1:56 PM smitra <smi...@zonnet.nl> wrote:
On 09-05-2022 00:34, Bruce Kellett wrote:

> That still treats the SE as indubitally true. No theory in physics is
> 'indubitably true'.
>
> The Everett program is to say that the SE is all that there is -- it
> explains everything. That is clearly false (no Born rule in the SE),
> so it might be wise to doubt the universal application of the SE.


There is no good reason to doubt the SE without any experimental hints
that it breaks down, or any good theoretical reasons why it is likely to
break down in some regime.

Such faith would be touching if it weren't so naive. There are good theoretical and experimental reasons to believe that it cannot be the whole story.

Bruce

smitra

unread,
May 11, 2022, 1:11:15 AM5/11/22
to everyth...@googlegroups.com
As John Clark has also mentioned, the opposite is true. There are no
good arguments for collapse theories. There are no experimental hints
for real collapse and if we argue based on theory, then we see that it
leads to many problems. Believing in collapse is like believing in the
ether after special relativity was already formulated and experimentally
confirmed.

Saibal



> Bruce
>
> --
> You received this message because you are subscribed to the Google
> Groups "Everything List" group.
> To unsubscribe from this group and stop receiving emails from it, send
> an email to everything-li...@googlegroups.com.
> To view this discussion on the web visit
> https://groups.google.com/d/msgid/everything-list/CAFxXSLSFh6-AjAE%3DbSQRnDFWJWOi4BVvEmFFv6WkvUD9FLJBgQ%40mail.gmail.com
> [1].
>
>
> Links:
> ------
> [1]
> https://groups.google.com/d/msgid/everything-list/CAFxXSLSFh6-AjAE%3DbSQRnDFWJWOi4BVvEmFFv6WkvUD9FLJBgQ%40mail.gmail.com?utm_medium=email&utm_source=footer

smitra

unread,
May 11, 2022, 1:16:15 AM5/11/22
to everyth...@googlegroups.com
There is no such thing as a mathematical continuum in the real physical
world. There are only a finite number of distinct quantum states
available for a finite universe. This is clear for states below some
total energy E. But there is an upper limit to the total energy due to
gravitational collapse when the energy exceeds a certain limit.


>> But one can also consider observers and then
>> each observer has a some finite memory so there are only a finite
>> number
>> of branches the observer can distinguish between.
>
> That does not follow.
>

If there are only a finite number of states the entire universe can be
in, then that's also true for observers.

Saibal

> Bruce
>
> --
> You received this message because you are subscribed to the Google
> Groups "Everything List" group.
> To unsubscribe from this group and stop receiving emails from it, send
> an email to everything-li...@googlegroups.com.
> To view this discussion on the web visit
> https://groups.google.com/d/msgid/everything-list/CAFxXSLQ%2B9Ot9VBtuwLdPSLAXbjSLdgvRdGo%3D%3DcDfbDNyia_N-Q%40mail.gmail.com
> [1].
>
>
> Links:
> ------
> [1]
> https://groups.google.com/d/msgid/everything-list/CAFxXSLQ%2B9Ot9VBtuwLdPSLAXbjSLdgvRdGo%3D%3DcDfbDNyia_N-Q%40mail.gmail.com?utm_medium=email&utm_source=footer

Bruce Kellett

unread,
May 11, 2022, 1:25:22 AM5/11/22
to Everything List
On Wed, May 11, 2022 at 3:11 PM smitra <smi...@zonnet.nl> wrote:
On 11-05-2022 06:06, Bruce Kellett wrote:
> On Wed, May 11, 2022 at 1:56 PM smitra <smi...@zonnet.nl> wrote:
>
>> On 09-05-2022 00:34, Bruce Kellett wrote:
>>
>>> That still treats the SE as indubitally true. No theory in physics is
>>> 'indubitably true'.
>>>
>>> The Everett program is to say that the SE is all that there is -- it
>>> explains everything. That is clearly false (no Born rule in the SE),
>>> so it might be wise to doubt the universal application of the SE.
>>
>> There is no good reason to doubt the SE without any experimental hints
>> that it breaks down, or any good theoretical reasons why it is
>> likely to break down in some regime.
>
> Such faith would be touching if it weren't so naive. There are good
> theoretical and experimental reasons to believe that it cannot be the
> whole story.
>
As John Clark has also mentioned, the opposite is true. There are no
good arguments for collapse theories. There are no experimental hints
for real collapse

That depends on how you read the data. We only see one outcome for each experiment, after all!
and if we argue based on theory, then we see that it
leads to many problems.

The SE also has many problems., as I have taken pains to point out.

Bruce

Brent Meeker

unread,
May 11, 2022, 1:25:39 AM5/11/22
to everyth...@googlegroups.com
Well, there's a big fat hint that it breaks down FAPP in every
measurement, in every bit of physics that appears classical and
irreversible.  So it has a burden to explain this appearance.  I see
some progress in this direction in decoherence and Zurek's quantum
Darwinism.  But it still ends in hand waving, "Since the SE applies to
everything the wave-function must be real and every component of it must
exist."  Which begs the question, "Does it apply to everything?"  It
doesn't apply to spacetime and gravity.  And it might just be an
effective approximation as in QBism.

Brent

Brent Meeker

unread,
May 11, 2022, 1:28:40 AM5/11/22
to everyth...@googlegroups.com
Quite aside from memory, per Everett there are a bazillion branches that
are only "measured" by the environment and no human is every aware of. 
But shall we not consider the "memory of the environment".  That's where
decoherence says the classical result gets recorded.

Brent

Bruce Kellett

unread,
May 11, 2022, 1:30:32 AM5/11/22
to Everything List
On Wed, May 11, 2022 at 3:16 PM smitra <smi...@zonnet.nl> wrote:
On 11-05-2022 06:01, Bruce Kellett wrote:
> On Wed, May 11, 2022 at 1:51 PM smitra <smi...@zonnet.nl> wrote:
>
>> On 09-05-2022 00:42, Bruce Kellett wrote:
>>>
>>> Such models are certainly inconsistent with the SE. So if your concern
>>> is that the SE does not contain provision for a collapse, then you
>>> should doubt other theories that violate the SE. You can't have it
>>> both ways: you can't reject collapse models because they violate the
>>> SE and then embrace other models that also violate the SE. Either the
>>> SE is universally correct, or it is not.
>>>
>>>> What matters is that such models can be
>>>> formulated in a mathematically consistent way, which demonstrates that
>>>> there is n o contradiction. The physical plausibility of such models
>>>> is another issue.
>>>

>>
>> As Brent has also pointed out, there amount of information in the
>> visible universe is finite.
>
> That does not limit the number of branches. A finite universe does not
> limit the number of points in a line.

There is no such thing as a mathematical continuum in the real physical
world.

Can you prove that? There is no evidence that space and time are discrete.


There are only a finite number of distinct quantum states
available for a finite universe.

Who proved that the universe was finite?

This is clear for states below some
total energy E. But there is an upper limit to the total energy due to
gravitational collapse when the energy exceeds a certain limit.


>> But one can also consider observers and then
>> each observer has a some finite memory so there are only a finite
>> number of branches the observer can distinguish between.
>
> That does not follow.
>

If there are only a finite number of states the entire universe can be
in, then that's also true for observers.

That simply begs the question.

Bruce

Brent Meeker

unread,
May 11, 2022, 1:39:45 AM5/11/22
to everyth...@googlegroups.com
So what does the SE for this discrete universe look like?  The one every
cites assumes a continuum.  If the universe is finite then there's
smallest non-zero probability,  which as Bruce says, raises some problems.

Brent

Brent Meeker

unread,
May 11, 2022, 1:42:39 AM5/11/22
to everyth...@googlegroups.com


On 5/10/2022 9:47 PM, smitra wrote:
> On 11-05-2022 06:06, Bruce Kellett wrote:
>> On Wed, May 11, 2022 at 1:56 PM smitra <smi...@zonnet.nl> wrote:
>>
>>> On 09-05-2022 00:34, Bruce Kellett wrote:
>>>
>>>> That still treats the SE as indubitally true. No theory in physics
>>> is
>>>> 'indubitably true'.
>>>>
>>>> The Everett program is to say that the SE is all that there is --
>>> it
>>>> explains everything. That is clearly false (no Born rule in the
>>> SE),
>>>> so it might be wise to doubt the universal application of the SE.
>>>
>>> There is no good reason to doubt the SE without any experimental
>>> hints
>>> that it breaks down, or any good theoretical reasons why it is
>>> likely to
>>> break down in some regime.
>>
>> Such faith would be touching if it weren't so naive. There are good
>> theoretical and experimental reasons to believe that it cannot be the
>> whole story.
>>
> As John Clark has also mentioned, the opposite is true. There are no
> good arguments for collapse theories. There are no experimental hints
> for real collapse

That's complete and audacious question begging.  What you mean by "real"
is "modeled within the SE".  There is NOTHING BUT collapse
experimentally; every result recorded in every notebook and every tape
is evidence of a collapse.

Brent

Bruce Kellett

unread,
May 11, 2022, 2:14:17 AM5/11/22
to Everything List
Not the least of these problems is the fact that a smallest non-zero probability makes the collapse real; destroys the ongoing superposition; renders everything absolutely irreversible; and screws the hell out of unitary evolution.

Bruce

John Clark

unread,
May 11, 2022, 6:50:41 AM5/11/22
to 'Brent Meeker' via Everything List
On Wed, May 11, 2022 at 1:25 AM Bruce Kellett <bhkel...@gmail.com> wrote:

> The SE also has many problems

The Schrodinger equation has ONE problem, SE can't account for gravity; General Relativity can but GR can't account for anything else.  Maybe when we find one physical idea that covers everything it will turn out that no quantum interpretation currently in use is correct, but until that time it's clear to me that Many Worlds is the least bad. I've said it before but I'll say it again, whatever turns out to be true one thing we can be certain of, it will be weird.

John K Clark    See what's on my new list at  Extropolis

wwe



John Clark

unread,
May 11, 2022, 7:27:59 AM5/11/22
to 'Brent Meeker' via Everything List
On Wed, May 11, 2022 at 1:25 AM Brent Meeker <meeke...@gmail.com> wrote:

> Well, there's a big fat hint that it [SE] breaks down FAPP in every
measurement, in every bit of physics that appears classical and
irreversible. 

The thing is, whenever somebody says FAPP they really don't mean for ALL practical purposes, certainly not if your practical purpose is to probe the fundamental nature of reality. In classical physics it's not literally impossible for the scrambled eggs on your breakfast plate to unscramble themselves before you have a chance to eat them, it's just very very unlikely. But if Many Worlds is correct then there are either an infinite number or an astronomical number to an astronomical power number of worlds where everything that is not forbidden to occur does occur, and one of the things that is not forbidden to occur is a world in which scrambled eggs unscramble themselves on your breakfast plate. If something happens a very very large number of times then even very very unlikely things will happen in some of them.  

  > And it might just be an effective approximation as in QBism.

QBism is not wrong, it's an effective way to do science if you're only interested in measurements and in making a new gadget that works and don't care about what's really going on; the same thing could be said about the Copenhagen Interpretation, and also for the all time favorite of most physicists, Shut Up And Calculate. But I find it extremely difficult to find any significant difference between QBism, Copenhagen and Shut Up And Calculate.
  John K Clark    See what's on my new list at  Extropolis 
suq

Brent Meeker

unread,
May 11, 2022, 12:43:33 PM5/11/22
to everyth...@googlegroups.com
As do other real-collapse theories of QM.

Brent

smitra

unread,
May 11, 2022, 4:08:46 PM5/11/22
to everyth...@googlegroups.com
And the results of those experiments lead to a theory where time
evolution is given by a unitary transform. It's as John Clark also
mentioned in one of his replies, analogous to how time reversal symmetry
is not apparent in the macroscopic world. But we know that the
fundamental laws are time reversible. This apparent discrepancy can be
explained, it's not evidence for time reversibility being violated in
nature.

>> and if we argue based on theory, then we see that it
>> leads to many problems.
>
> The SE also has many problems., as I have taken pains to point out.
>

There are no problems with the SE. It's not inconsistent with the Born
rule. The only issue is that it looks a bit unnatural for a fundamental
law of physics to require both a dynamical ruke and the Born rule. But a
real collapse is inconsistent with the SE.

Saibal

> Bruce
>
> --
> You received this message because you are subscribed to the Google
> Groups "Everything List" group.
> To unsubscribe from this group and stop receiving emails from it, send
> an email to everything-li...@googlegroups.com.
> To view this discussion on the web visit
> https://groups.google.com/d/msgid/everything-list/CAFxXSLQtFVcW_4fSQjOrHCu-atrTi4SnXH8KbbVXYRVSP%2B6PVg%40mail.gmail.com
> [1].
>
>
> Links:
> ------
> [1]
> https://groups.google.com/d/msgid/everything-list/CAFxXSLQtFVcW_4fSQjOrHCu-atrTi4SnXH8KbbVXYRVSP%2B6PVg%40mail.gmail.com?utm_medium=email&utm_source=footer

Brent Meeker

unread,
May 11, 2022, 5:02:33 PM5/11/22
to everyth...@googlegroups.com
Not in QBism.  It's just updating your prior.  Seems a perfect fit for
someone who wants to take an information theoretic approach and model
consciousness as an algorithm.

Brent

smitra

unread,
May 11, 2022, 5:33:20 PM5/11/22
to everyth...@googlegroups.com
You then have a finite set of states with transition probabilities for
transitions between the states.

Saibal

> Brent

smitra

unread,
May 11, 2022, 5:43:37 PM5/11/22
to everyth...@googlegroups.com
It does not have the burden to explain this fully. Lacking a good
explanation, one has to go through all reasonable explanations based on
what is known. A good example is what John Clark also mentioned about
time reversibility. Boltzmann presented an incomplete argument on how
the tension between the Second law and microscopic time reversibility
could be resolved. And while the precise rigorous argument was not
satisfactorily settled until quite recently, it was good enough for
physicists to move on and accept that the laws of physics are time
reversible.

This is because it was implausible to have time reversibility at the
micro-level and not at a higher level, given that what happens at the
macro-level is fully determined by what happens at the micro-level. It
would require new experimental results to cast doubt on reversibility,
so the burden of proof is on anyone proposing such a hypothesis.

The case of unitary time evolution is similar. There are no experimental
results that demonstrate that this is violated. While we do have
effective collapse at the macro-scale, this is what one would expect due
to decoherence.


> I see
> some progress in this direction in decoherence and Zurek's quantum
> Darwinism.  But it still ends in hand waving,

Physics would not be possible without lots of hand waving.


> "Since the SE applies to
> everything the wave-function must be real and every component of it
> must exist." 
> Which begs the question, "Does it apply to everything?" 
> It doesn't apply to spacetime and gravity.

It does apply to spacetime, at least there is no problem with assuming
that it does. See e.g. here for some details:

https://arxiv.org/abs/1407.4748

In particular Section 4.1. "How to Make Computations" on page 23 and
further


>   And it might just be an
> effective approximation as in QBism.
>

That could be, but as things stand now, there is no evidence for that.

Saibal

> Brent

smitra

unread,
May 11, 2022, 6:36:26 PM5/11/22
to everyth...@googlegroups.com
There is effective collapse in experiments we do, but the experiments
nevertheless demonstrate that the fundamental processes proceed under
unitary time evolution.

Saibal

Brent Meeker

unread,
May 11, 2022, 6:44:28 PM5/11/22
to everyth...@googlegroups.com


On 5/11/2022 1:06 PM, smitra wrote:
That's complete and audacious question begging.  What you mean by
"real" is "modeled within the SE".  There is NOTHING BUT collapse
experimentally; every result recorded in every notebook and every tape
is evidence of a collapse.


There is effective collapse in experiments we do, but the experiments nevertheless demonstrate that the fundamental processes proceed under unitary time evolution.

Except when you measure them and actually get a result.

Brent

Bruce Kellett

unread,
May 11, 2022, 7:01:34 PM5/11/22
to Everything List
On Thu, May 12, 2022 at 8:36 AM smitra <smi...@zonnet.nl> wrote:
On 11-05-2022 07:42, Brent Meeker wrote:

> That's complete and audacious question begging.  What you mean by
> "real" is "modeled within the SE".  There is NOTHING BUT collapse
> experimentally; every result recorded in every notebook and every tape
> is evidence of a collapse.
>

There is effective collapse in experiments we do, but the experiments
nevertheless demonstrate that the fundamental processes proceed under
unitary time evolution.

All that the experiments demonstrate is that the wave function evolves unitarily between state preparation and measurement. This is most easily accounted for by assuming that the wave function is a purely epistemic vehicle for the time evolution of probabilities. Since it is purely epistemic, collapse is not a problem since it is not a physical event. One does not have to go the whole way to QBism -- the wave function can still be objective (inter-subjectively agreed).

Bruce

smitra

unread,
May 11, 2022, 7:08:08 PM5/11/22
to everyth...@googlegroups.com
Counterexample: The internal state of an ideal quantum computer will
always evolve under unitary time evolution.

Saibal

> Bruce
>
> --
> You received this message because you are subscribed to the Google
> Groups "Everything List" group.
> To unsubscribe from this group and stop receiving emails from it, send
> an email to everything-li...@googlegroups.com.
> To view this discussion on the web visit
> https://groups.google.com/d/msgid/everything-list/CAFxXSLTj9WvFA-MHW%3DCPKAPzBKPWgnsNDxZe-QiHrMXmMH4PTQ%40mail.gmail.com
> [1].
>
>
> Links:
> ------
> [1]
> https://groups.google.com/d/msgid/everything-list/CAFxXSLTj9WvFA-MHW%3DCPKAPzBKPWgnsNDxZe-QiHrMXmMH4PTQ%40mail.gmail.com?utm_medium=email&utm_source=footer

smitra

unread,
May 11, 2022, 7:16:35 PM5/11/22
to everyth...@googlegroups.com
Yes, but note that the observer's memory then factors out of the part of
the state that descibes what the observer is not aware of, and that
part is then in a superposition of all possibilities. See also here:

https://arxiv.org/abs/1804.00749

"A no-go theorem for observer-independent facts"

Saibal

> Brent

smitra

unread,
May 11, 2022, 7:24:28 PM5/11/22
to everyth...@googlegroups.com
In physics we only have a continuum in the scaling limit where we've
scaled the microscopic distances away to zero. Whenever we do a
computations where it really matters whether or not the continuum is
real, we end up having to impose a short-ditance cut-off and can only
remove this cut-off via a renormalization procedure. See also page 12 of
this document:

https://webspace.science.uu.nl/~hooft101/lectures/basisqft.pdf

"Often, authors forget to mention the first, very important, step in
this logical procedure: replace the classical field theory one wishes to
quantize by a strictly finite theory.
Assuming that physical structures smaller than a certain size will not
be important for
our considerations, we replace the continuum of three-dimensional space
by a discrete but
dense lattice of points. In the differential equations, we replace all
derivatives ∂/∂xi by
finite ratios of differences: ∆/∆x
, where ∆φ stands for φ(x + ∆x) − φ(x) . In Fourier
space, this means that wave numbers ~k are limited to a finite range
(the Brillouin zone),
so that integrations over ~k can never diverge.
If this lattice is sufficiently dense, the solutions we are interested
in will hardly depend
on the details of this lattice, and so, the classical system will resume
Lorentz invariance
and the speed of light will be the practical limit for the velocity of
perturbances. If
necessary, we can also impose periodic boundary conditions in 3-space,
and in that case
our system is completely finite. Finite systems of this sort allow for
‘quantization’ in the
old-fashioned sense: replace the Poisson brackets by commutators. "


>
>> There are only a finite number of distinct quantum states
>> available for a finite universe.
>
> Who proved that the universe was finite?
>

If it's infinite, one can focus on only the visible part of it.

>> This is clear for states below some
>> total energy E. But there is an upper limit to the total energy due
>> to
>> gravitational collapse when the energy exceeds a certain limit.
>>
>>>> But one can also consider observers and then
>>>> each observer has a some finite memory so there are only a finite
>>>> number of branches the observer can distinguish between.
>>>
>>> That does not follow.
>>>
>>
>> If there are only a finite number of states the entire universe can
>> be
>> in, then that's also true for observers.
>
> That simply begs the question.
>

Finite or infinite universe, observers are always finite.

Saibal

> Bruce
>
> --
> You received this message because you are subscribed to the Google
> Groups "Everything List" group.
> To unsubscribe from this group and stop receiving emails from it, send
> an email to everything-li...@googlegroups.com.
> To view this discussion on the web visit
> https://groups.google.com/d/msgid/everything-list/CAFxXSLR%2Bakx3k3W3QXHHPCgnYaUZ%3Dp6m8uoxS0RKiyp8Xaw6oQ%40mail.gmail.com
> [1].
>
>
> Links:
> ------
> [1]
> https://groups.google.com/d/msgid/everything-list/CAFxXSLR%2Bakx3k3W3QXHHPCgnYaUZ%3Dp6m8uoxS0RKiyp8Xaw6oQ%40mail.gmail.com?utm_medium=email&utm_source=footer

Bruce Kellett

unread,
May 11, 2022, 7:36:52 PM5/11/22
to Everything List
On Thu, May 12, 2022 at 9:24 AM smitra <smi...@zonnet.nl> wrote:
On 11-05-2022 07:30, Bruce Kellett wrote:
>
> Who proved that the universe was finite?
>

If it's infinite, one can focus on only the visible part of it.

The visible part is only locally defined -- go to the edge and there is another, larger, region.

>> If there are only a finite number of states the entire universe can
>> be in, then that's also true for observers.
>
> That simply begs the question.
>

Finite or infinite universe, observers are always finite.

The universe itself is not defined by observers.

Bruce

Bruce Kellett

unread,
May 11, 2022, 7:47:09 PM5/11/22
to Everything List
On Thu, May 12, 2022 at 9:08 AM smitra <smi...@zonnet.nl> wrote:
On 11-05-2022 08:14, Bruce Kellett wrote:
> On Wed, May 11, 2022 at 3:39 PM Brent Meeker <meeke...@gmail.com>
> wrote:
>
>> On 5/10/2022 9:43 PM, smitra wrote:
>>
>>> If there are only a finite number of states the entire universe can be
>>> in, then that's also true for observers.
>>
>> So what does the SE for this discrete universe look like?  The one
>> every cites assumes a continuum.  If the universe is finite then there's
>> smallest non-zero probability,  which as Bruce says, raises some
>> problems.
>
> Not the least of these problems is the fact that a smallest non-zero
> probability makes the collapse real; destroys the ongoing
> superposition; renders everything absolutely irreversible; and screws
> the hell out of unitary evolution.


Counterexample: The internal state of an ideal quantum computer will
always evolve under unitary time evolution.

If there is a smallest non-zero probability, this may no longer be the case. Actually, a smallest non-zero probability would certainly resolve a lot of the problems with many worlds theory. Unitarity would no longer work to all levels; pure states would automatically become mixtures under decoherence; reversibility would vanish; collapse would make sense, and the emergence of the classical world from the underlying quantum substrate would be explained. All this follows if there are no continuous quantities in physics, and continuous variables are just approximations to underlying discrete quantities......

Solves a lot of problems. I can see why Brent is attracted to this idea.

Bruce

smitra

unread,
May 12, 2022, 3:12:46 PM5/12/22
to everyth...@googlegroups.com
A real collapse is nevertheless inconsistent with the SE, there would
exist physical processes where the SE would fail. If real collapse is
supposed to happen in experiments, then because experiments are
ultimately just many particle interactions then that means that, in
general, the SE cannot be exactly valid. We may then try to observe
small violations of the SE in the lab.

Saibal

> Brent

smitra

unread,
May 12, 2022, 3:22:06 PM5/12/22
to everyth...@googlegroups.com
No, there exist no experiment results that demonstrate that unitary time
evolution is not exactly valid. What you are referring to is that in
experiments we do the wavefunction of the measured system (effectively)
collapses. But, because we also know from all the experimental results
that the wavefunction evolves in a unitary way, and experiments are
ultimately nothing more that many particle interactions, that either
unitary time evolution cannot be exactly valid or that the collapse
during measurement is an artifact of decoherence where the observer (and
the local environment) gets into an entangled superposition with the
measured system. The former hypothesis lacks experimental support.

Saibal


> Brent
>
> --
> You received this message because you are subscribed to the Google
> Groups "Everything List" group.
> To unsubscribe from this group and stop receiving emails from it, send
> an email to everything-li...@googlegroups.com.
> To view this discussion on the web visit
> https://groups.google.com/d/msgid/everything-list/4824477e-6952-5c26-4b4f-00be790f6b50%40gmail.com
> [1].
>
>
> Links:
> ------
> [1]
> https://groups.google.com/d/msgid/everything-list/4824477e-6952-5c26-4b4f-00be790f6b50%40gmail.com?utm_medium=email&utm_source=footer

smitra

unread,
May 12, 2022, 3:37:01 PM5/12/22
to everyth...@googlegroups.com
That's possible but that means that QM is not a complete fundamental
theory of reality. Anything that explains these probabilities is then
possible, including the existence of a multiverse.

Saibal

> Bruce
>
> --
> You received this message because you are subscribed to the Google
> Groups "Everything List" group.
> To unsubscribe from this group and stop receiving emails from it, send
> an email to everything-li...@googlegroups.com.
> To view this discussion on the web visit
> https://groups.google.com/d/msgid/everything-list/CAFxXSLSf3z54_UNEvZBRsazzYFPWPx-GaBq3xY8UyDZATsArCQ%40mail.gmail.com
> [1].
>
>
> Links:
> ------
> [1]
> https://groups.google.com/d/msgid/everything-list/CAFxXSLSf3z54_UNEvZBRsazzYFPWPx-GaBq3xY8UyDZATsArCQ%40mail.gmail.com?utm_medium=email&utm_source=footer

smitra

unread,
May 12, 2022, 3:57:38 PM5/12/22
to everyth...@googlegroups.com
On 12-05-2022 01:36, Bruce Kellett wrote:
> On Thu, May 12, 2022 at 9:24 AM smitra <smi...@zonnet.nl> wrote:
>
>> On 11-05-2022 07:30, Bruce Kellett wrote:
>>>
>>> Who proved that the universe was finite?
>>>
>>
>> If it's infinite, one can focus on only the visible part of it.
>
> The visible part is only locally defined -- go to the edge and there
> is another, larger, region.
>

Yes, but in the end this doesn't really matter due to there only being
local interactions. After a finite time any finite system can only
interact with a finite number of degrees of freedom in its environment.

>>>> If there are only a finite number of states the entire universe
>> can
>>>> be in, then that's also true for observers.
>>>
>>> That simply begs the question.
>>>
>>
>> Finite or infinite universe, observers are always finite.
>
> The universe itself is not defined by observers.
>

The state of the observer can then factor out of the branches the
universe is in.

Saibal

> Bruce
>
> --
> You received this message because you are subscribed to the Google
> Groups "Everything List" group.
> To unsubscribe from this group and stop receiving emails from it, send
> an email to everything-li...@googlegroups.com.
> To view this discussion on the web visit
> https://groups.google.com/d/msgid/everything-list/CAFxXSLT_m8o6hx%2BNCkFg9rALPXYozuWgwVMGL-okgY4RtX6KnA%40mail.gmail.com
> [1].
>
>
> Links:
> ------
> [1]
> https://groups.google.com/d/msgid/everything-list/CAFxXSLT_m8o6hx%2BNCkFg9rALPXYozuWgwVMGL-okgY4RtX6KnA%40mail.gmail.com?utm_medium=email&utm_source=footer

smitra

unread,
May 12, 2022, 4:03:37 PM5/12/22
to everyth...@googlegroups.com
A quantum computer implements exactly the sort of a discrete system that
is discussed, and yet it works just fine, evolving under the unitary
time evolutions as it should during the time it can be maintained in a
quantum coherent state.


> Actually, a smallest non-zero probability would certainly
> resolve a lot of the problems with many worlds theory. Unitarity would
> no longer work to all levels; pure states would automatically become
> mixtures under decoherence; reversibility would vanish; collapse would
> make sense, and the emergence of the classical world from the
> underlying quantum substrate would be explained. All this follows if
> there are no continuous quantities in physics, and continuous
> variables are just approximations to underlying discrete
> quantities......
>
> Solves a lot of problems. I can see why Brent is attracted to this
> idea.
>

This does not follow from the non-existence of continuous quantities,
because nothing on the current laws of physics implies that continuous
quantities objectively exist.

> Bruce
>
> --
> You received this message because you are subscribed to the Google
> Groups "Everything List" group.
> To unsubscribe from this group and stop receiving emails from it, send
> an email to everything-li...@googlegroups.com.
> To view this discussion on the web visit
> https://groups.google.com/d/msgid/everything-list/CAFxXSLQ1J0gJy2F1EVCZA8BuiO-2ae7RDc9xcVrWPCVWZ%3DYTpA%40mail.gmail.com
> [1].
>
>
> Links:
> ------
> [1]
> https://groups.google.com/d/msgid/everything-list/CAFxXSLQ1J0gJy2F1EVCZA8BuiO-2ae7RDc9xcVrWPCVWZ%3DYTpA%40mail.gmail.com?utm_medium=email&utm_source=footer

Brent Meeker

unread,
May 12, 2022, 4:18:56 PM5/12/22
to everyth...@googlegroups.com
I agree.  And in fact SE fails all the time.  It fails to predict a
definite outcome...which is OK if you accept probabilistic theories. 
But then its real failure is that it doesn't tell you exactly when and
where and why it stops unitary evolution and produces a result.  The
Born rule tells us the probability of a result...IF there is one. 
Decoherence tells there's an asymptotic approach to a result and
why...but not when and where it arrives.

Brent

Brent Meeker

unread,
May 12, 2022, 4:23:59 PM5/12/22
to everyth...@googlegroups.com


On 5/12/2022 11:27 AM, smitra wrote:
> On 12-05-2022 00:44, Brent Meeker wrote:
>> On 5/11/2022 1:06 PM, smitra wrote:
>>
>>>> That's complete and audacious question begging.  What you mean by
>>>> "real" is "modeled within the SE".  There is NOTHING BUT collapse
>>>> experimentally; every result recorded in every notebook and every
>>>> tape
>>>> is evidence of a collapse.
>>>
>>> There is effective collapse in experiments we do, but the
>>> experiments nevertheless demonstrate that the fundamental processes
>>> proceed under unitary time evolution.
>>
>> Except when you measure them and actually get a result.
>>
>
> No, there exist no experiment results that demonstrate that unitary
> time evolution is not exactly valid. What you are referring to is that
> in experiments we do the wavefunction of the measured system
> (effectively) collapses. But, because we also know from all the
> experimental results that the wavefunction evolves in a unitary way,
> and experiments are ultimately nothing more that many particle
> interactions, that either unitary time evolution cannot be exactly
> valid or that the collapse during measurement is an artifact of
> decoherence where the observer (and the local environment) gets into
> an entangled superposition with the measured system. The former
> hypothesis lacks experimental support.

"we also know from all the experimental results that the wavefunction
evolves in a unitary way"...until we get a result and then it doesn't.

So does the latter.  It's based purely on the absence of a theory of
collapse, beyond, perhaps, decoherence which provides a (sort of) theory
of pointer basis and approach to collapse.

Brent

Brent Meeker

unread,
May 12, 2022, 4:27:26 PM5/12/22
to everyth...@googlegroups.com


On 5/12/2022 11:42 AM, smitra wrote:
All that the experiments demonstrate is that the wave function evolves
unitarily between state preparation and measurement. This is most
easily accounted for by assuming that the wave function is a purely
epistemic vehicle for the time evolution of probabilities. Since it is
purely epistemic, collapse is not a problem since it is not a physical
event. One does not have to go the whole way to QBism -- the wave
function can still be objective (inter-subjectively agreed).


That's possible but that means that QM is not a complete fundamental theory of reality. Anything that explains these probabilities is then possible, including the existence of a multiverse.

Which is about as explanatory as "God did it."  Explaining the values of the probabilities isn't the problem with MWI,  it's explaining that there are probabilities even though nothing happens, and when and where the probabilities arise.

Brent

Brent Meeker

unread,
May 12, 2022, 4:39:53 PM5/12/22
to everyth...@googlegroups.com
But it doesn't give an answer by evolving unitarily.

>
>
>> Actually, a smallest non-zero probability would certainly
>> resolve a lot of the problems with many worlds theory. Unitarity would
>> no longer work to all levels; pure states would automatically become
>> mixtures under decoherence; reversibility would vanish; collapse would
>> make sense, and the emergence of the classical world from the
>> underlying quantum substrate would be explained. All this follows if
>> there are no continuous quantities in physics, and continuous
>> variables are just approximations to underlying discrete
>> quantities......
>>
>> Solves a lot of problems. I can see why Brent is attracted to this
>> idea.
>>
>
> This does not follow from the non-existence of continuous quantities,
> because nothing on the current laws of physics implies that continuous
> quantities objectively exist.

All the more reason to suspect that there is a smallest non-zero
probability.

Brent

Bruce Kellett

unread,
May 12, 2022, 8:50:58 PM5/12/22
to Everything List
On Fri, May 13, 2022 at 5:57 AM smitra <smi...@zonnet.nl> wrote:
On 12-05-2022 01:36, Bruce Kellett wrote:
> On Thu, May 12, 2022 at 9:24 AM smitra <smi...@zonnet.nl> wrote:
>
>> On 11-05-2022 07:30, Bruce Kellett wrote:
>>> Who proved that the universe was finite?
>>
>> If it's infinite, one can focus on only the visible part of it.
>
> The visible part is only locally defined -- go to the edge and there
> is another, larger, region.
>

Yes, but in the end this doesn't really matter due to there only being
local interactions. After a finite time any finite system can only
interact with a finite number of degrees of freedom in its environment.

But that does not mean that variables are discrete rather than continuous.
>>>> If there are only a finite number of states the entire universe can
>>>> be in, then that's also true for observers.
>>>
>>> That simply begs the question.
>>>
>>
>> Finite or infinite universe, observers are always finite.
>
> The universe itself is not defined by observers.


The state of the observer can then factor out of the branches the
universe is in.

That is just a meaningless contention. The state of the observer, or what the observer is aware of, or can or cannot factor out, is irrelevant to the universe. Reality is not defined by observers.

Bruce

Bruce Kellett

unread,
May 12, 2022, 8:58:58 PM5/12/22
to Everything List
On Fri, May 13, 2022 at 5:22 AM smitra <smi...@zonnet.nl> wrote:
On 12-05-2022 00:44, Brent Meeker wrote:
> On 5/11/2022 1:06 PM, smitra wrote:
>
>> There is effective collapse in experiments we do, but the
>> experiments nevertheless demonstrate that the fundamental processes
>> proceed under unitary time evolution.
>
> Except when you measure them and actually get a result.
>

No, there exist no experiment results that demonstrate that unitary time
evolution is not exactly valid. What you are referring to is that in
experiments we do the wavefunction of the measured system (effectively)
collapses. But, because we also know from all the experimental results
that the wavefunction evolves in a unitary way, and experiments are
ultimately nothing more that many particle interactions, that either
unitary time evolution cannot be exactly valid or that the collapse
during measurement is an artifact of decoherence where the observer (and
the local environment) gets into an entangled superposition with the
measured system. The former hypothesis lacks experimental support.

The multiverse hypothesis also lacks experimental support. We observe collapse every day and in every experiment. We never observe a multiverse.

Bruce

John Clark

unread,
May 13, 2022, 8:09:25 AM5/13/22
to 'Brent Meeker' via Everything List
On Thu, May 12, 2022 at 4:27 PM Brent Meeker <meeke...@gmail.com> wrote:

> Explaining the values of the probabilities isn't the problem with MWI,  it's explaining that there are probabilities

That's easy in MWI. Probabilities exist because until you actually look at it there is no way to know if you are the Brent Meeker who lives in a universe where the electron went left or you are the Brent Meeker who lives in a universe where the electron went right, due to the fact that the only difference between the two Brent Meekers is what the electron does.
 
  John K Clark    See what's on my new list at  Extropolis
mbe


smitra

unread,
May 13, 2022, 3:24:06 PM5/13/22
to everyth...@googlegroups.com
As I wrote in the previous reply, physics does not work in the way you
are arguing here. You always have to formulate a well defined set of
hypotheses first which you can then test with experimental data. There
are plenty of examples where people tried to do test in a supposedly
model independent way and then got results that were not at all model
independent.

In this case, that the wavefunction collapses or at least appears to, is
something that's treated radically different between the CI-like
hypotheses and the MWI-like hypotheses. So, we can consider a class of
MWI-like theories where there is no collapse with CI-like theories where
there is collapse and then consider how they explain all of the
experimental data.

If you do that, then you see that CI-like theories postulate a new
physical mechanism for collapse that's left unspecified that cannot be
explained from the interaction Hamiltonian that one uses. Here I'm
staying within the context of the CI, I'm not introducing any baggage
from the MWI.

In MWI-like theories, there is nothing else than what is described by
the interaction Hamiltonian. The problem here is to get to a better
explanation of ho decoherence leads to the effective classical world.

The former problem is a real physics problem where one depends on a new
phenomena, just like e.g. dark matter in cosmology. It has to exist
according to the theory, but it hasn't yet been discovered yet. But
unlike in case of dark matter where there are multiple independent
observational results that point to its existence, in case of collapse,
you only have the mere fact that in experiments the wavefunction
collapses.

The problems with MWI-like theories is usual business that's seen in
most other theories. Take e.g. superconductivity and we have plenty of
experimental data that's not well explained yet by the theory. But this
does not lead physicists to postulate new physics.

Saibal




> Brent

smitra

unread,
May 13, 2022, 3:36:09 PM5/13/22
to everyth...@googlegroups.com
> there ARE probabilities even though nothing happens, and when and
> where the probabilities arise.
>

I agree with what John Clark said in his reply.

To add to that, the "God did it" thing applies far more to the CI,
because there one postulates the collapse without explaining the
mechanism for it. In the MWI one assumes that the appearance of collapse
can be explained from the known dynamics. Those explanations may not be
satisfactory as of yet, but that's typical for most of science. There
are phenomena that as of yet are not well explained, but that does not
(necessarily) lead us to postulate new physics all the time. Doing so
would make us like creationists who tend to invoke a "God of the gaps".

Saibal


> Brent
>
> --
> You received this message because you are subscribed to the Google
> Groups "Everything List" group.
> To unsubscribe from this group and stop receiving emails from it, send
> an email to everything-li...@googlegroups.com.
> To view this discussion on the web visit
> https://groups.google.com/d/msgid/everything-list/03b48558-b7a4-a48a-ee68-58e5e07931a4%40gmail.com
> [1].
>
>
> Links:
> ------
> [1]
> https://groups.google.com/d/msgid/everything-list/03b48558-b7a4-a48a-ee68-58e5e07931a4%40gmail.com?utm_medium=email&utm_source=footer

smitra

unread,
May 13, 2022, 3:36:20 PM5/13/22
to everyth...@googlegroups.com
True, but as long as we (can) keep it isolated, the evolution is
unitary, and that contradicts the point Brice was making about discrete
systems and unitary time evolution.

>>
>>
>>> Actually, a smallest non-zero probability would certainly
>>> resolve a lot of the problems with many worlds theory. Unitarity
>>> would
>>> no longer work to all levels; pure states would automatically become
>>> mixtures under decoherence; reversibility would vanish; collapse
>>> would
>>> make sense, and the emergence of the classical world from the
>>> underlying quantum substrate would be explained. All this follows if
>>> there are no continuous quantities in physics, and continuous
>>> variables are just approximations to underlying discrete
>>> quantities......
>>>
>>> Solves a lot of problems. I can see why Brent is attracted to this
>>> idea.
>>>
>>
>> This does not follow from the non-existence of continuous quantities,
>> because nothing on the current laws of physics implies that continuous
>> quantities objectively exist.
>
> All the more reason to suspect that there is a smallest non-zero
> probability.

Yes, but the point Brice makes about unitary time evolution is not true,
at least not in the general way he formulated it.

Saibal
>
> Brent

smitra

unread,
May 13, 2022, 3:40:24 PM5/13/22
to everyth...@googlegroups.com
On 13-05-2022 02:50, Bruce Kellett wrote:
> On Fri, May 13, 2022 at 5:57 AM smitra <smi...@zonnet.nl> wrote:
>
>> On 12-05-2022 01:36, Bruce Kellett wrote:
>>> On Thu, May 12, 2022 at 9:24 AM smitra <smi...@zonnet.nl> wrote:
>>>
>>>> On 11-05-2022 07:30, Bruce Kellett wrote:
>>>>> Who proved that the universe was finite?
>>>>
>>>> If it's infinite, one can focus on only the visible part of it.
>>>
>>> The visible part is only locally defined -- go to the edge and
>> there
>>> is another, larger, region.
>>>
>>
>> Yes, but in the end this doesn't really matter due to there only
>> being
>> local interactions. After a finite time any finite system can only
>> interact with a finite number of degrees of freedom in its
>> environment.
>
> But that does not mean that variables are discrete rather than
> continuous.
>

I agree, not by itself.

>>>>>> If there are only a finite number of states the entire universe
>> can
>>>>>> be in, then that's also true for observers.
>>>>>
>>>>> That simply begs the question.
>>>>>
>>>>
>>>> Finite or infinite universe, observers are always finite.
>>>
>>> The universe itself is not defined by observers.
>>
>> The state of the observer can then factor out of the branches the
>> universe is in.
>
> That is just a meaningless contention. The state of the observer, or
> what the observer is aware of, or can or cannot factor out, is
> irrelevant to the universe. Reality is not defined by observers.
>

I fully agree. But this is precisely an argument in favor of the
multiverse when applied to the different sectors.

Saibal

> Bruce
>
> --
> You received this message because you are subscribed to the Google
> Groups "Everything List" group.
> To unsubscribe from this group and stop receiving emails from it, send
> an email to everything-li...@googlegroups.com.
> To view this discussion on the web visit
> https://groups.google.com/d/msgid/everything-list/CAFxXSLRSiX0gaOj7mG7U0pObfB99FrrwVLCYWsuCSKmbkpWV4Q%40mail.gmail.com
> [1].
>
>
> Links:
> ------
> [1]
> https://groups.google.com/d/msgid/everything-list/CAFxXSLRSiX0gaOj7mG7U0pObfB99FrrwVLCYWsuCSKmbkpWV4Q%40mail.gmail.com?utm_medium=email&utm_source=footer

smitra

unread,
May 13, 2022, 3:42:51 PM5/13/22
to everyth...@googlegroups.com
Which is consistent with the multiverse hypothesis.

Saibal

> Bruce
>
> --
> You received this message because you are subscribed to the Google
> Groups "Everything List" group.
> To unsubscribe from this group and stop receiving emails from it, send
> an email to everything-li...@googlegroups.com.
> To view this discussion on the web visit
> https://groups.google.com/d/msgid/everything-list/CAFxXSLTFptj7ttYYvfyP6A2Ep%3DMernnzMR4Znmx4Xk6-Nxh4NA%40mail.gmail.com
> [1].
>
>
> Links:
> ------
> [1]
> https://groups.google.com/d/msgid/everything-list/CAFxXSLTFptj7ttYYvfyP6A2Ep%3DMernnzMR4Znmx4Xk6-Nxh4NA%40mail.gmail.com?utm_medium=email&utm_source=footer

smitra

unread,
May 13, 2022, 3:43:55 PM5/13/22
to everyth...@googlegroups.com
On 13-05-2022 14:08, John Clark wrote:
> On Thu, May 12, 2022 at 4:27 PM Brent Meeker <meeke...@gmail.com>
> wrote:
>
>> _> Explaining the values of the probabilities isn't the problem with
>> MWI, it's explaining that there ARE probabilities_
>
> That's easy in MWI. Probabilities exist because until you actually
> look at it there is no way to know if you are the Brent Meeker who
> lives in a universe where the electron went left or you are the Brent
> Meeker who lives in a universe where the electron went right, due to
> the fact that the only difference between the two Brent Meekers is
> what the electron does.
>

Indeed!

Saibal

> John K Clark See what's on my new list at Extropolis [1]
>
> mbe
>
> --
> You received this message because you are subscribed to the Google
> Groups "Everything List" group.
> To unsubscribe from this group and stop receiving emails from it, send
> an email to everything-li...@googlegroups.com.
> To view this discussion on the web visit
> https://groups.google.com/d/msgid/everything-list/CAJPayv3Dn8ghJUMTt%3DU0L48ROnNYOpweXCa1sB0os140U8FNyA%40mail.gmail.com
> [2].
>
>
> Links:
> ------
> [1] https://groups.google.com/g/extropolis
> [2]
> https://groups.google.com/d/msgid/everything-list/CAJPayv3Dn8ghJUMTt%3DU0L48ROnNYOpweXCa1sB0os140U8FNyA%40mail.gmail.com?utm_medium=email&utm_source=footer

smitra

unread,
May 13, 2022, 3:51:21 PM5/13/22
to everyth...@googlegroups.com
Physics doesn't work in this way. You always need to define a well
defined hypothesis first in order to interpret experimental results and
be able to test various alternative hypotheses/theories. If you don't do
this, you are not doing physics.

 
> But then its real failure is that it doesn't tell you exactly when and
> where and why it stops unitary evolution and produces a result. 

That's a failure of particular interpretations of QM, e.g. the CI that
postulate collapse.

> The
> Born rule tells us the probability of a result...IF there is one. 
> Decoherence tells there's an asymptotic approach to a result and
> why...but not when and where it arrives.

Decoherence does does tell you how the different sectors split over
time.

Saibal

>
> Brent

Brent Meeker

unread,
May 13, 2022, 3:59:54 PM5/13/22
to everyth...@googlegroups.com
Right CI doesn't explain the collapse and MWI doesn't explain the
collapse either but assumes it can be explained without new physics.  I
hypothesize (not assume) that CI+ <non-zero minimum probability> can
explain the collapse.  I don't see any big advantage for MWI here.  My
attitude toward interpretations is that they are unimportant in
themselves, but they are useful in pointing to new, more comprehensive
and accurate theories.  That's one reason I'm not impressed by MWI since
it seems to ex hypothesi put any emprical testing out of reach.

Brent

Brent Meeker

unread,
May 13, 2022, 4:06:43 PM5/13/22
to everyth...@googlegroups.com
Which is why assuming the SE is the whole truth even though it predicts
that everything possible happens, isn't doing physics.

Brent

Bruce Kellett

unread,
May 13, 2022, 9:07:05 PM5/13/22
to Everything List
On Sat, May 14, 2022 at 5:51 AM smitra <smi...@zonnet.nl> wrote:
On 12-05-2022 22:18, Brent Meeker wrote:
>
> I agree.  And in fact SE fails all the time.  It fails to predict a
> definite outcome...which is OK if you accept probabilistic theories.

Physics doesn't work in this way. You always need to define a well
defined hypothesis first in order to interpret experimental results and
be able to test various alternative hypotheses/theories. If you don't do
this, you are not doing physics.

Tell that to the army of people who pounce on every anomaly that appears in analyses of partial data from the LHC or Tevatron. Every anomaly produces a slew of papers, all proposing "explanations" of the anomaly. This is an industry, it is not physics. Generally the anomalies go away with time and further data -- there are no "well defined hypotheses" at work here.


> But then its real failure is that it doesn't tell you exactly when and
> where and why it stops unitary evolution and produces a result. 

That's a failure of particular interpretations of QM, e.g. the CI that
postulate collapse.

> The Born rule tells us the probability of a result...IF there is one. 
> Decoherence tells there's an asymptotic approach to a result and
> why...but not when and where it arrives.

Decoherence does does tell you how the different sectors split over
time.

Not if unitary evolution is exact and always. You have often argued that the original superposition never really goes away. Strictly, that means that the initial state is still intact, and nothing has in fact happened. Decoherence has to work through to a conclusion if the sectors are to split and a definite result is to emerge. This is where unitary evolution breaks down. Taken literally it never leads to a result. Just as in a quantum computer -- the internal unitary evolution has to invoke decoherence and collapse in order for a result to emerge.

You need some marker of the point at which the different sectors finally differentiate. The SE itself is clearly not the whole story.......you need something like a minimum non-zero probability! Or an acceptance that FAPP is good enough, along with an understanding of when FAPP is good enough.

Bruce

Stathis Papaioannou

unread,
May 13, 2022, 9:46:04 PM5/13/22
to everyth...@googlegroups.com
On Fri, 13 May 2022 at 22:09, John Clark <johnk...@gmail.com> wrote:

On Thu, May 12, 2022 at 4:27 PM Brent Meeker <meeke...@gmail.com> wrote:

> Explaining the values of the probabilities isn't the problem with MWI,  it's explaining that there are probabilities

That's easy in MWI. Probabilities exist because until you actually look at it there is no way to know if you are the Brent Meeker who lives in a universe where the electron went left or you are the Brent Meeker who lives in a universe where the electron went right, due to the fact that the only difference between the two Brent Meekers is what the electron does.

But you don’t think this applies with non MWI duplication.

  John K Clark    See what's on my new list at  Extropolis
mbe


--
You received this message because you are subscribed to the Google Groups "Everything List" group.
To unsubscribe from this group and stop receiving emails from it, send an email to everything-li...@googlegroups.com.
--
Stathis Papaioannou

John Clark

unread,
May 13, 2022, 10:06:26 PM5/13/22
to 'Brent Meeker' via Everything List
On Fri, May 13, 2022 at 9:46 PM Stathis Papaioannou <stat...@gmail.com> wrote:

>>> Explaining the values of the probabilities isn't the problem with MWI,  it's explaining that there are probabilities

>> That's easy in MWI. Probabilities exist because until you actually look at it there is no way to know if you are the Brent Meeker who lives in a universe where the electron went left or you are the Brent Meeker who lives in a universe where the electron went right, due to the fact that the only difference between the two Brent Meekers is what the electron does.

> But you don’t think this applies with non MWI duplication.

That is simply NOT true! After my body has been duplicated but before I have open the door of the duplicating chamber to see where I was I won't know if I will be the John Clark who has seen Moscow or the John Clark who has seen Helsinki, and indeed the distinction between the two would be meaningless because the two would be identical until the door is opened and they differentiate because then one has the memory of seeing Moscow but the other has the memory of seeing Helsinki.  So if both decided to place a bet on what they would see after the door was opened (and if one decided to place a bet then the other certainly would too because they're identical) then, provided they were logical,and I think I am at least most of the time, they would both put the odds at 50-50.

  John K Clark    See what's on my new list at  Extropolis
lmt


It is loading more messages.
0 new messages